寄托家园留学论坛

标题: argument同主题写作第二类 8月16-17日 [打印本页]

作者: tesolchina    时间: 2017-8-11 12:45:01     标题: argument同主题写作第二类 8月16-17日

本帖最后由 tesolchina 于 2017-8-18 11:54 编辑

第二组的argument同主题练习,只涉及较少的题目,训练的重点是写开头段和中间段。
1. 请观看Lecture 5 Writing Introductory and body paragraphs for GRE arguments
视频链接

2. 选取第二类argument的其中一篇完成claim-data-warrant分析
3. 根据以上的分析及Lecture 5的内容写出完整的开头段和一个中间段。

Feedback up to Floor no. 43
作者: tesolchina    时间: 2017-8-12 12:06:52

Arg 1 Palean Basket unique

Woven baskets characterized by a particular distinctive pattern have previously been found only in the immediate vicinity of the prehistoric village of Palea and therefore were believed to have been made only by the Palean people. Recently, however, archaeologists discovered such a 'Palean' basket in Lithos, an ancient village across the Brim River from Palea. The Brim River is very deep and broad, and so the ancient Paleans could have crossed it only by boat, and no Palean boats have been found. Thus it follows that the so-called Palean baskets were not uniquely Palean.

Write a response in which you discuss what specific evidence is needed to evaluate the argument and explain how the evidence would weaken or strengthen the argument.

53) The following appeared in a health magazine.

The citizens of Forsythe have adopted more healthful lifestyles. Their responses to a recent survey show that in their eating habits they conform more closely to government nutritional recommendations than they did ten years ago. Furthermore, there has been a fourfold increase in sales of food products containing kiran, a substance that a scientific study has shown reduces cholesterol. This trend is also evident in reduced sales of sulia, a food that few of the most healthy citizens regularly eat.

Write a response in which you discuss what specific evidence is needed to evaluate the argument and explain how the evidence would weaken or strengthen the argument.

144) The citizens of Forsythe have adopted more healthful lifestyles. Their responses to a recent survey show that in their eating habits they conform more closely to government nutritional recommendations than they did ten years ago. Furthermore, there has been a fourfold increase in sales of food products containing kiran, a substance that a scientific study has shown reduces cholesterol. This trend is also evident in reduced sales of sulia, a food that few of the healthiest citizens regularly eat.

Write a response in which you examine the stated and/or unstated assumptions of the argument. Be sure to explain how the argument depends on these assumptions and what the implications are for the argument if the assumptions prove unwarranted.

151) Benton City residents have adopted healthier lifestyles. A recent survey of city residents shows that the eating habits of city residents conform more closely to government nutritional recommendations than they did ten years ago. During those ten years, local sales of food products containing kiran, a substance that a scientific study has shown reduces cholesterol, have increased fourfold, while sales of sulia, a food rarely eaten by the healthiest residents, have declined dramatically. Because of these positive changes in the eating habits of Benton City residents, we predict that the obesity rate in the city will soon be well below the national average.

Write a response in which you discuss what questions would need to be answered in order to decide whether the prediction and the argument on which it is based are reasonable. Be sure to explain how the answers to these questions would help to evaluate the prediction.
作者: 东扯扯    时间: 2017-8-16 10:10:24

1)
Woven baskets characterized by a particular distinctive pattern have previously been found only in the immediate vicinity of the prehistoric village of Palea and therefore were believed to have been made only by the Palean people. Recently, however, archaeologists discovered such a 'Palean' basket in Lithos, an ancient village across the Brim River from Palea. The Brim River is very deep and broad, and so the ancient Paleans could have crossed it only by boat, and no Palean boats have been found. Thus it follows that the so-called Palean baskets were not uniquely Palean.

Write a response in which you discuss what specific evidence is needed to evaluate the argument and explain how the evidence would weaken or strengthen the argument.

claim: woven baskets have been made only by the Palean people
data: woven baskets have previously been found only in the immediate vicinity of the prehistoric village of Palea
warrant: the place where they have previously found woven baskets must be the origin
evidence: 1. the correlation between the particular pattern of the basket and the local culture and ritual; 2. the word or picture records in Palea such as hieroglyph describing the process of making the basket

claim: ancient Paleans could not cross the Brim River
data: the river is so broad that someone can cross it only by boat which have not been found in Palea
warrant: there was no boat in Palea and the river at that time could cross only by boat
evidence: 1. other river-crossing methods, such as the vestige of bridge; 2. geographical conditions in the ancient about the river (width and depth)

claim: the so-called Palean baskets were not uniquely Palean
data: a such Palean basket was discovered in Litho
warrant: there was no way to send basket from Palean to Litho
evidence: 1. other methods at that time to send basket from Palean to Litho; 2. related records about the transportation of this kind of basket


Intro
In this argument, it is concluded that the baskets which were previously only found in Palea were not only belong to Palean people, since that a same basket was also found in Lithos and the possibility that this basket found in Lithos was from Palea has been ruled out because of the broad and wide river between them. However, in order to fully evaluate this argument, it requires a significant amount of additional evidence about the associate records, geographical and weather conditions at that time and vestige of river-crossing method. With the extra evidence, the argument could end up being much weaker than it seems, or it might actually be quite valid.



Ts2:
It is assumed that the basket found in Lithos could not be from Palea because there was no way to cross the Brim River based on given evidence. However, in view of the limited information, there are other possible scenarios. For example, the river between these two places at that time might not as the same as what it is now. Geographical conditions about the Brim River in the ancient is required to assess the argument. If the river was so narrow and shallow that people could easily cross it on foot or even there was no river at all, the argument would be groundless. Even if there was a broad and deep river, the evidence of prestige of bridge by which people could cross this river could likewise weaken the argument.


作者: wanghaitao19939    时间: 2017-8-16 14:13:02

Arg 1 Palean Basket unique

Woven baskets characterized by a particular distinctive pattern have previously been found only in the immediate vicinity of the prehistoric village of Palea and therefore were believed to have been made only by the Palean people. Recently, however, archaeologists discovered such a 'Palean' basket in Lithos, an ancient village across the Brim River from Palea. The Brim River is very deep and broad, and so the ancient Paleans could have crossed it only by boat, and no Palean boats have been found. Thus it follows that the so-called Palean baskets were not uniquely Palean.

Write a response in which you discuss what specific evidence is needed to evaluate the argument and explain how the evidence would weaken or strengthen the argument.

Claim: Palean baskets were not uniquely Palean.
Data: Recently, however, archaeologists discovered such a 'Palean' basket in Lithos, an ancient village across the Brim River from Palea. The Brim River is very deep and broad, and so the ancient Paleans could have crossed it only by boat, and no Palean boats have been found.
Warrant: so-called Palean baskets were developed independently in both Palea and Lithos.

Evidence:
1.Same origin.
There may be earthquakes producing a broad river cut Palea into two villages, so the same pattern on woven baskets can be found in Lithos across the Brim River.
2.Land modificaiton.
It is possible that the Brim River was not as deep and broad as it is today when people in Palea and Lithos could easily communicate the same pattern on woven baskets. So they didn’t use boat to go across the river.
3.More archaeology evidence about boat.
No Palean boats have been found doesn’t mean that the boats were not exist, further archaeology found should be done.

开头段:
According to the passage, the Palean baskets were not uniquely Palean. Because archaeologists discovered such a 'Palean' basket in Lithos, a village across the Brim River from Palea. Since the Brim River is too deep and broad to be crossed and no Palean boats have been found, so-called Palean baskets were developed independently in both Palea and Lithos. There are some evidence should be used to scrutinize the argument critically, to assess the strength of the argument.
中间段:
To begin with, the statement indicates that archaeologists discovered such a 'Palean' basket in Lithos, a village across also the Brim River from Palea would lead to two different origins of particular distinctive pattern on woven baskets. Neglecting land modification, it assumes that the land form near Palea has not changed after the emerging of particular distinctive pattern on woven baskets. While it is possible that a earthquake produced a broad river cut Palea into two villages, so the same pattern on woven baskets can be found in Lithos across the Brim River. If the land modification be proved by further archaeology evidence, the statement will be weakened.

In addition, overlooking water fluctuation, the argument assumes that the river has not changed in so many years. We need to take the possibility of fluctuation of the Brim River into account. It is possible that the Brim River was not as deep and broad as it is today when people could easily deliver the pattern on woven baskets to Lithos from Palea. To go across a shallow river, the ancient people didn’t need boat. If more geographic evidence be evacuated, this assumption may be questioned.

Another assumption in the argument is no Palean boats have been found means the boats were not exist nor the communication between Palea and Lithos. As we all know that in ancient society the boats were made of wood which is easy to be destroyed, if there were boats used by ancient people around the Palean and Lithos not been found, the communication between Palea and Lithos should not be denied so imprudently.
结尾:
In conclusion, it is too abrupt to assert the Palean baskets were not uniquely Palean. The assumptions discussed above should be critically examined by taking into account more archaeology evidence such as the land modification, water fluctuation and boat residues.  
感觉用来论证的专门用语还是不够多,来回就这么两种句式,我都觉得自己像个机器人,写的干巴巴。也有逻辑方面的缺点吧,继续练吧。
作者: kyika    时间: 2017-8-16 14:26:33

本帖最后由 kyika 于 2017-8-17 20:38 编辑

151) Benton City residents have adopted healthier lifestyles. A recent survey of city residents shows that the eating habits of city residents conform more closely to government nutritional recommendations than they did ten years ago. During those ten years, local sales of food products containing kiran, a substance that a scientific study has shown reduces cholesterol, have increased fourfold, while sales of sulia, a food rarely eaten by the healthiest residents, have declined dramatically. Because of these positive changes in the eating habits of Benton City residents, we predict that the obesity rate in the city will soon be well below the national average.

Write a response in which you discuss what questions would need to be answered in order to decide whether the prediction and the argument on which it is based are reasonable. Be sure to explain how the answers to these questions would help to evaluate the prediction.

A recent survey of city residents shows that the eating habits of city residents conform more closely to government nutritional recommendations than they did ten years ago.
Claim; Benton City residents have adopted healthier lifestyles
data: the eating habits of city residents conform more closely to government nutritional recommendations than they did ten years ago.
warrant: (more closely to government nutritional recommendations means healthier.)
Question: Whether the eating habits of citizens ten years ago healthier than the government nutritional recommendations or not?
Evidence: information about whether the government nutritional recommendations is the healthiest eating style.
Assumption:  Getting more close to government nutritional recommendations means healthier.

During those ten years, local sales of food products containing kiran, a substance that a scientific study has shown reduces cholesterol, have increased fourfold, while sales of sulia, a food rarely eaten by the healthiest residents, have declined dramatically.
claim: Benton City residents’ eating habits are becoming healthier.
data: local sales of food products containing kiran have increased fourfold. Sales of sulia have declined dramatically.
warrant: kiran and sulia can reflect the consumption of all the healthy and unhealthy food.
Question: Is other healthy food becoming more welcome? Is other unhealthy food becoming less welcome?
Assumption: Benton City residents are eating more healthy food and less unhealthy food than ten years ago.
Evidence: Information about the sale of all the healthy and unhealthy food.

Because of these positive changes in the eating habits of Benton City residents, we predict that the obesity rate in the city will soon be well below the national average.
Claim: obesity rate in the city will soon be well below the national average
data: Benton City residents’ eating habits are healthier than before.
warrant: Health eating habits can decrease obesity rate.
Question:         Is the obesity caused by unhealthy eating habits? Is there any other important factor relative to obesity? Is healthy eating habits effective to treat obesity?
Evidence: The reason of high obesity rate.
Assumption: High obesity rate is caused by unhealthy eating habits and can be cured by changing eating habits.

According to author, the obesity rate of Benton city will soon below average owing to the healthier eating style citizens adopt. Citing to comparison of citizens’ eating habits to government recommendations and data about sales of kiran and sulia, the writer try to prove it. A number of questions should be answered in order to evaluate the argument, such as whether the government nutritional recommendations was healthier than the eating habits ten years ago, whether the survey is reliable , whether healthy food becomes more welcome recently and what caused the obesity rate above average. Without the answer to questions above, we can’t come to the conclusion that the obesity rate will drop in the near future.

To begin with, we need to know whether Benton city residents are adopting healthier eating habits than before. Only that they are eating healthier, we can believe that maybe the obesity rate will decrease. Though the survey claim that their eating habits conform more closely to government nutritional recommendations, we don’t know the details of recommendations so that we can’t draw  a conclusion that more close to it means healthier. In addition, we want to ask whether the eating habits ten years ago is less healthy than the recommendations. If not, that survey argued that eating habits becoming more closely to the recommendations actually means they are eating less healthy and we still can’t come to a conclusion of decrease of obesity rate. Given the recommendations are standard of healthy diet and eating habits ten years ago are less healthy than it, the author need to answer questions about the reliability about the survey, such as how many residents are being investigate, can the participants reflect the eating habits on average and how to decide whether their eating habits is close to recommendations. Without the answers to these question, one can’t rely on the survey and conclude that residents are eating healthier.






东扯扯同学的习作点评
1. 您的开头段写的不错,很有条理,使用了连接词since however; 第一句话概括了conclusion & evidence, 解释了evidence怎样支持conclusion,我要向你学习。
2. 关于第一个claim-data-warrant的讨论
     您的第一个claim-data –warant 是针对第一句“Woven baskets characterized by a particular distinctive pattern have previously been found only in the immediate vicinity of the prehistoric village of Palea and therefore were believed to have been made only by the Palean people.”进行分析的,由于今天下午我也写过这篇argument的分析,我当时也有这种想法,但我后来觉得第一句是交代背景,是一些公认的事实,作者的陈述是基于这些事实的,所以“质疑basket是不是palean的”可能不妥?(个人观点)
3. 写作指导要求evidence,所以是不是把第二段开头It is assumed that 换成evidence about … is required之类的更能回应题目要求?


感谢woaka同学点评,很有帮助,习作已改(绿色)

作者: amber_123    时间: 2017-8-16 16:12:25

Arg 1 Palean Basket unique

Woven baskets characterized by a particular distinctive pattern have previously been found only in the immediate vicinity of the prehistoric village of Palea and therefore were believed to have been made only by the Palean people. Recently, however, archaeologists discovered such a 'Palean' basket in Lithos, an ancient village across the Brim River from Palea. The Brim River is very deep and broad, and so the ancient Paleans could have crossed it only by boat, and no Palean boats have been found. Thus it follows that the so-called Palean baskets were not uniquely Palean.

Write a response in which you discuss what specific evidence is needed to evaluate the argument and explain how the evidence would weaken or strengthen the argument.

1
Data: an ancient village across the Brim River from Palea, The Brim River is very deep and broad
Claim: ancient Paleans could have crossed it only by boat
Warrant: only by boat can paleans pass across the river

Evidence: no other possible solutions, like ice

2
data: no Palean boats have been found
claim: paleans haven’t invented boat at that time
warrant: if you can’t find boat now, it doesn’t exist at that time

evidence: no records of boat in other documents like writings, paintings, maybe people just haven’t find them now

3
data: Woven baskets characterized by a particular distinctive pattern have previously been found only in the immediate vicinity of the prehistoric village of Palea
Claim: and therefore were believed to have been made only by the Palean people.
Warrant: where the baskets are found, whom they are invented by

Evidence: other documents to prove the baskets are invented by paleans, maybe they are invented by other people, people today haven’t found yet

Introductory paragraph

According to the statement, Palean baskets are not uniquely Palean because archaeologists have found similar ones in Lithos. Given the fact that there’s a deep and broad river between Palea and Lithos and scientists have found no remaining of Palean boats, they think Lithoses invented the same baskets as Paleans. However, a number of evidences are missing related to the cross-river method, boat and basket inventor. These evidences need to be provided comprehensively in order to make the statement grounding.

Middle paragraph
To begin with, it’s been taken for granted that Paleans need to take a boat to go across the Brim River since it’s deep and broad. However, Paleans might possibly have other cross-river methods. It’s so cold in Palea in winter that the river’s got frozen. Paleans could step on the ice and walk across the river, which permits Paleans visit Lithos once a year to exchange merchandise like baskets. Evidences unable to exclude this possibility are weak to make us believe that Palean baskets are not unique by Paleans.

作者: 拒绝失学    时间: 2017-8-16 16:39:37

144) The citizens of Forsythe have adopted more healthful lifestyles. Their responses to a recent survey show that in their eating habits they conform more closely to government nutritional recommendations than they did ten years ago. Furthermore, there has been a fourfold increase in sales of food products containing kiran, a substance that a scientific study has shown reduces cholesterol. This trend is also evident in reduced sales of sulia, a food that few of the healthiest citizens regularly eat.

Write a response in which you examine the stated and/or unstated assumptions of the argument. Be sure to explain how the argument depends on these assumptions and what the implications are for the argument if the assumptions prove unwarranted.

Claim: The citizens of Forsythe have adopted more healthful lifestyles.
Data1: Their responses to a recent survey show that in their eating habits they conform more closely to government nutritional recommendations than they did ten years ago.
Warrant1: The respondents are representative to the whole residents of Forsythe, and their eating habits really conform to their reply.

Data2: there has been a fourfold increase in sales of food products containing kiran, a substance that a scientific study has shown reduces cholesterol.
Warrant2: Food products containing kiran are necessarily healthy, not containing other harmful ingredients. The increase of this kind of food is due to the more consumption of local citizens. People buy this food is out of the reason that it is healthy.

Data3: reduced sales of sulia, a food that few of the healthiest citizens regularly eat.
Warrant3: Few heathiest citizens regularly eat means sulia is unhealthy. The reduced sales of sulia is out of the reason that local people think it is unhealthy. This trend is stable and sustainable.

开头:
The author of the article holds the claim that the citizens of Forsythe have adopted more healthful lifestyles. According to the article, such a claim is based on three evidences, a survey, the increase of sales of a seemingly healthy kind of food, and reduced sales of an unhealthy food. Admittedly, the claim does seem compelling on the surface. However, if we put a further consideration upon the assumptions made by the author, we will find that there are several holes in the deduction. Also, if the author does not give more details, then the claim is untenable.

中间段:
The author mentioned the reduced sales of sulia as a main support for the claim, because sulia is rarely consumed by the healthiest citizens, thus unhealthy. Yet, such a fact does not necessarily imply unhealthiness. It is possible that sulia is relatively expensive relative to other healthy food, therefore few of consumers will choose it as a regular consumption. Even if sulia is indeed unhealthy, the author still does not give us the real reason regarding the reduced sales. Possibly, the majority of sulia consumers are tourists, instead of local residents, and the reduced sales is out of the decrease of tourists. Or, the increasing price prevents people from buying sulia. Plus, whether such a trend is stable and sustainable is unclear according to the passage. Only if people continuously buying less sulia can the claim be true. Therefore, the author should clarify these assumptions in a further way.
作者: mzlgjg    时间: 2017-8-16 17:18:48

53)The following appeared in a health magazine.

The citizens of Forsythe have adopted more healthful lifestyles. Their responses to a recent survey show that in their eating habits they conform more closely to government nutritional recommendations than they did ten years ago. Furthermore, there has been a fourfold increase in sales of food products containing kiran, a substance that a scientific study has shown reduces cholesterol. This trend is also evident in reduced sales of sulia, a food that few of the most healthy citizens regularly eat.

Write a response in which you discuss what specific evidence is needed to evaluate the argument and explain how the evidence would weaken or strengthen the argument.

Claim: Eating habits of citizens of F are conformed more closely to government nutritional recommendations.
Data: A recent survey shows that.
Warrant: This survey has the validity to comprehensively represent the eating habits of the majority of citizens of F.
Evidence: the validity of the survey

Claim: Eating habits of citizens are improved by a fourfold increase in food products containing kiran.
Data: There has been a fourfold increase in sales of food products containing kiran and scientific study has shown it reduces cholesterol.
Warrant: The increase of sale of food, the consumption of the food containing kiran will help citizens reduce their cholesterol to a healthier level.
Evidence: the relationship between increase of kiran and the reduction of cholesterol and the relationship between people’s health conditions and their cholesterol level.

Claim: Eating less sulia will strengthen people’s health.
Data: This trend is also evident in reduced sales of sulia.
Warrant: Sulia will not improve people’s health.
Evidence: Sulia will not improve people’s health.

Claim: Citizens achieve healthier life styles now.
Data: the following three above
Warrant: Healthier life styles is made through improvement of eating habits.

Introductory paragraph:
  The health magazine announces that life styles of citizens of Forsythe are better than before. Employing survey and investigation of sales of food in market, the magazine reaches this conclusion. Several evidence is needed in developing this argument about this survey and the relationship between certain food and people’s health. These evidence needs to be provided to fully validate this argument.

Middle paragraph:TS2
  It is also assumed that a fourfold more consumption of food products containing kiran will help people reduce their cholesterol level and become more healthier. However, evidence is needed to provide for how much cholesterol kiran can reduce and relationship between cholesterol level attuned by kiran and people’s health. So it still remains an open question whether this increase consumption of food containing kiran will reduce cholesterol to a healthier level or not. If this increase consumption will not reduce people’s cholesterol to a healthier level, then the argument made by the magazine is weakened.
作者: amber_123    时间: 2017-8-16 17:57:05

53) The following appeared in a health magazine.

The citizens of Forsythe have adopted more healthful lifestyles. Their responses to a recent survey show that in their eating habits they conform more closely to government nutritional recommendations than they did ten years ago. Furthermore, there has been a fourfold increase in sales of food products containing kiran, a substance that a scientific study has shown reduces cholesterol. This trend is also evident in reduced sales of sulia, a food that few of the most healthy citizens regularly eat.

Write a response in which you discuss what specific evidence is needed to evaluate the argument and explain how the evidence would weaken or strengthen the argument.

1
data: Their responses to a recent survey show that in their eating habits they conform more closely to government nutritional recommendations than they did ten years ago.
Claim: The citizens of Forsythe have adopted more healthful lifestyles.
Warrant: nutritional eating habits mean healthful lifestyles

Evidences: other aspects of lifestyles like exercise, mental health

2
data: there has been a fourfold increase in sales of food products containing kiran, a substance that a scientific study has shown reduces cholesterol.
Claim: The citizens of Forsythe have adopted more healthful lifestyles.
Warrant: increasing sales of healthy food proves that citizens adopt healthful lifestyles

Evidences: 1) no other influential factors like they like the flavor, price, good promotion
2) the residents’ health condition after the increase (do less people catch cholesterol)

3
data: This trend is also evident in reduced sales of sulia, a food that few of the most healthy citizens regularly eat
claim: The citizens of Forsythe have adopted more healthful lifestyles.
Warrant: healthy citizens don’t eat sulia because it’s unhealthy

Evidences: sulia is unhealthy (don’t like the flavor, price)

Introductory paragraph

According to the magazine, the residents of Forsythe have changed to healthier lifestyles by consuming recommended nutrition, eating healthy food which contains kiran and eating less unhealthy food like sulia. However, a number of evidences are missing related to the other aspects of residents’ lifestyles, the reason why residents choose food with kiran and the reason why they dislike sulia. These evidences need to be provided in order to scrutinize the statement and thus make it grounding.

Middle paragraph

To begin with, it is taken for granted that the residents are living healthier lifestyles because their eating habit is consistent with government nutritional recommendations. However, evidences of other aspects of residents’ lifestyles are not available. Though they eat healthier, they may go to gym less. They would suffer from big working pressure. Lacking of exercises and suffering from pressure are not signs of healthy lifestyles. Evidences of other aspects of lifestyles are important for us to determine whether the residents are living healthier lifestyles except their eating habit.

作者: Angelyyy    时间: 2017-8-16 18:50:46

1. Palean Basket unique

Woven baskets characterized by a particular distinctive pattern have previously been found only in the immediate vicinity of the prehistoric village of Palea and therefore were believed to have been made only by the Palean people. Recently, however, archaeologists discovered such a 'Palean' basket in Lithos, an ancient village across the Brim River from Palea. The Brim River is very deep and broad, and so the ancient Paleans could have crossed it only by boat, and no Palean boats have been found. Thus it follows that the so-called Palean baskets were not uniquely Palean.

Write a response in which you discuss what specific evidence is needed to evaluate the argument and explain how the evidence would weaken or strengthen the argument.

Claim(C):Palean baskets were not unique in Palea.
Data/Facts(D/F):
1.Both Palea and Lithos found the basket.
2.The Brim River which split up the two villages which people couldn’t across without a boat.
3.There are no boats found in Palea.
Warrant:
1.Archeaologists found such a ”Palean” basket in Lithos.

Assumption:
1.The Palean could only arrive Lithos by crossing the river.
2.Only the Palean can arrive Lithos,things couldn’t be vice versa.
Questions Evidence needed:
-Whether there are other paths could let the Palean accessible to the Lithos?Possibility could be that at the age of the ancient period,the river is not so deep that people could cross it easily without trouble?We need the historical description to make it clear.
-If the archaeologists discovered boats in Lithos?If the possibility is people in LIthos learned the skill from the Palea crossing the river by boat?Maybe the relationship between the two villages is very harmonious that they could communicate with each other.
-After all,the possibility could be that the boat is deteriorated completely even though they once there.

According to the description of the research of the Palea basket,the author made the conclusion that the basket is not unique in Palea.However,as far as I am concerned,it would be untenable to concede that.For the assumption made by the description,it would be presumptuous to say the river is so deep that people can’t cross without a boat.I doubt that if the river is as deep as the author depicted at that time.On the other hand,whether there are boats in Lithos?We should not to ignore the possibility that people in Lithos could cross the river to learn the skill from the Palea.And I would address the details of my arguments.

Let’s see the first doubt that if the river is so deep at the ancient time.While we should admit the fact that people is hard to cross the river unless the assistant of boats,however,we couldn’t ignore the accuracy of the historical document of the depth of the river.The possibility could be that the river is shallow and could let the villagers walk cross easily.If the communication is possible,we couldn’t make the decision of the origin of the basket.Thus,it’s better to display the accurate documents of the depth of the river at that time to support the conclusion the essay given

Turn our attention to the boats.Providing that the description is true that there is no boats find in the Paea area,how about the villagers in Lithos cross the river to learn the skills from the Palea?The Possibility could be that the communication between the two villages is harmonious and the Lithos always ride the boats to visit their friendly neighborhood in Palea.This could be a reasonable explanation of the basket found in Lithos.As a result,we need the evidence of the archaeologists’ exploration or related academic research to make sure if there is any sign to show the Lithos people once took any boats cross the river.  

Excepting the possibilities I mentioned,there is also could doubt if is tenable to believe there is no boats remained in the Palea instead of the deterioration effect of the boats?We need the related historical documents to make it clear.

点评板凳楼:
我觉得写的挺好的呀,我也想到了ancient time不见得river就不让人过,必须要有船,我只想到了precipitation,水不够深,你还想到了地震的原因,棒!
另外我也怀疑了文章说找不到船这个事实的真实性,握爪。
另外,the conclusion be weakened,be questioned这样的表达挺好的。
最后,有一些语法小错误你自己回头放在word里查一下就好啦。

作者: 卿本悠悠    时间: 2017-8-16 19:09:44

本帖最后由 卿本悠悠 于 2017-8-16 19:12 编辑

5、more healthful lifestyles(144)

The citizens of Forsythe have adopted more healthful lifestyles. Their responses to a recent survey show that in their eating habits they conform more closely to government nutritional recommendations than they did ten years ago. Furthermore, there has been a fourfold increase in sales of food products containing kiran, a substance that a scientific study has shown reduces cholesterol. This trend is also evident in reduced sales of sulia, a food that few of the healthiest citizens regularly eat.

Write a response in which you examine the stated and/or unstated assumptions of the argument. Be sure to explain how the argument depends on these assumptions and what the implications are for the argument if the assumptions prove unwarranted.


Claim: their eating habits becomes more healthier
Data: Their responses to a recent survey show that in their eating habits they conform more closely to government nutritional recommendations than they did ten years ago.
Warrant: they say that their eating habits are more close to government nutritional recommendations than ten years ago
Assumption:
1. What they said to the survey was true and nobody or few person lied in their response.
2. The healthiness of government nutritional recommendations to humans are verified and supported by academic authority

Claim: they eat much more healthy food products
Data: there has been a fourfold increase in sales of food products containing kiran, a substance that a scientific study has shown reduces cholesterol.
Warrant: these food products containing kiran which can reduce cholesterol
Assumption:
1. Citizens purchase the food products for their own daily eating, instead of for pets or some certain patients in need
2. The results of scientific study about kiran are academic valid
3. That reducing cholesterol leads to healthful lifestyles is real.

Claim: citizens less buy unhealthy food
Data: This trend is also evident in reduced sales of sulia, a food that few of the healthiest citizens regularly eat.
Warrant: Citizens reduce the purchase of sulia, which healthiest people rarely buy
Assumption:
1. The sulia is evidently unhealthy
2. What the healthiest citizens seldom eat is truly unhealthy

Introductory paragraph
Based on a recent survey, the citizens of Forsythe have taken more healthy lifestyles than ten years ago. By summarizing and analyzing these citizens’ responses, the survey shows the transition of their eating habits in many aspects. A number of assumptions has been put forward in the argument about the survey, the certain food and the citizens. These assumptions need to be carefully examined to prove the preciseness of the argument result.

Middle paragraphs (the first)
First of all, it should be assumed that what citizens of Forsythe respond to the survey is true and nobody or few person have lied in their response. If the reality of their responses couldn’t be ensured, results based on the survey are invalid and meaningless, which means no need to check and analyze any subsequent outcome. Moreover, it is also assumed that the healthiness of government nutritional recommendations to humans are verified and supported by academic authority. Maybe these recommendations are designed according to many extra considerations beside healthiness, like the convenience for food, the tradition and social economy. There might exists healthier eating habits which are different or contrary to the government nutritional recommendations.

作者: 战G进行时    时间: 2017-8-16 20:14:24

53) The following appeared in a health magazine.

The citizens of Forsythe have adopted more healthful lifestyles. Their responses to a recent survey show that in their eating habits they conform more closely to government nutritional recommendations than they did ten years ago. Furthermore, there has been a fourfold increase in sales of food products containing kiran, a substance that a scientific study has shown reduces cholesterol. This trend is also evident in reduced sales of sulia, a food that few of the most healthy citizens regularly eat.

Write a response in which you discuss what specific evidence is needed to evaluate the argument and explain how the evidence would weaken or strengthen the argument.


Claim: The citizens of Forsythe have adopted more healthful lifestyles.
Data: In their eating habits they conform more closely to government nutritional recommendations than they did ten years ago.
Warrant: Nutrition is helpful to keep them healthy.

Claim: The citizens of Forsythe have adopted more healthful lifestyles.
Data: There has been a fourfold increase in sales of food products containing kiran, a substance that a scientific study has shown reduces cholesterol.
Warrant: The increase of sales of food products containing kiran indicate that people adopted a healthier diet.

Claim: The citizens of Forsythe have adopted more healthful lifestyles.
Data: The reduction of the sales of sulia is evident, a food that few of the most healthy citizens regularly eat.
Warrant: The decrease of sales of sulia indicate that people adopted a healthier diet.

1st paragraph
According to the description in the health magazine, a healthier lifestyle has been accommodated by the citizens of Forsythe. Through citing the result of a recent survey concerning the fool consumption of the citizens, the essay reaches the conclusion. A number of assumptions have been made in the argument about the reliability of the survey and the relation between the lifestyle and their food consumption. These assumptions need to be critically examined to assess the strength of the argument.

Middle paragraph
The conclusion is based on the assumption that the results of the survey is reliable. However, in the essay, there is no statistics to show how many citizens join in the survey. Moreover, there is no evidence to prove that the participants is representative enough to indicate the conditions of all the citizens of Forsythe. Perhaps most of the participants are the elders or women who focus more on healthy lifestyle. If the participants are not randomly picked out, representative of all the citizens, then the result of the survey is not reliable. As a result, the conclusion that they adopt a more heathful lifestyle is not credible.

作者: 改了名字你们就认不出我了    时间: 2017-8-16 21:04:37

1)
Woven baskets characterized by a particular distinctive pattern have previously been found only in the immediate vicinity of the prehistoric village of Palea and therefore were believed to have been made only by the Palean people. Recently, however, archaeologists discovered such a 'Palean' basket in Lithos, an ancient village across the Brim River from Palea. The Brim River is very deep and broad, and so the ancient Paleans could have crossed it only by boat, and no Palean boats have been found. Thus it follows that the so-called Palean baskets were not uniquely Palean.

Write a response in which you discuss what specific evidence is needed to evaluate the argument and explain how the evidence would weaken or strengthen the argument.


Claim: The so-called Palean baskets were not uniquely Palean.
Data: Archaeologists discovered such a 'Palean' basket in Lithos, an ancient village across the Brim River from Palea.
Warrant: The baskets found in Lithos were local product.
Evidence needed: People in Lithos are able to produce such kind of basket.

Claim: it’s impossible for the Paleans to cross the river
Data: The Brim River is very deep and broad and no Palean boats have been found
Warrant: the ancient Paleans could have crossed it only by boat
Evidence needed: 1. the river is deep and broad in the ancient times.
                2. the boats that could be used to cross the river never existed

Claim: Woven baskets were believed to have been made only by the Palean people.
Data: Woven baskets characterized by a particular distinctive pattern have previously been found only in the immediate vicinity of the prehistoric village of Palea.
Warrant: Palean people made these woven baskets.
Evidence needed: Palean people are isolated.


开头段:

According to the passage above, the conclusion that the so-called Palean basket were not uniquely Palean is reached by two steps. Firstly, recent discovery of a ‘Palean’ basket in Lithos represents that the woven baskets are not uniquely Palean. Secondly, due to the deep and broad river between these two villages, the basket found in Lithos is impossible to be made in Palea. While the conclusion could be true, close scrutiny reveals that the conclusion lacks critical support and therefore we need more evidence to help evaluate the argument.

中间段:
First of all, a recent discovery of a ‘Palean’ basket in Lithos is given to suggest that the so-called Palean baskets were possible to exist outside Palean. However, we need more evidence to ascertain whether the ‘Palean’ basket in Lithos are indeed local product. First, we need to find out whether the materials used to produce woven basket is available in Lithos. Since materials are the fundamental part of composition, only when the abundant resources of fundamental part are proved to exist, can we take the latter whole of basket into consideration. Second, we need some traces to identify that people in Lithos have mastered the craft to make woven basket. Such traces consist of a laundry list of tools used during the process of making and preserving woven baskets. If the aforementioned evidence can be found, the claim that the woven basket could be made in Lithos will be more plausible. Otherwise, the conclusion will be weakened.

In addition, we need more evidence to ascertain that baskets found in Lithos were not delivered from Palea. There are two facts given to support the claim: the river between these two villages is too broad and deep to cross without boat and no Palean boats have been found. However, the reasoning process of these facts are not flawless. For example, it’s possible that, in ancient times, the Brim river was not wide and deep as we see today which would be easy for the Palean to cross. Also, the non-existence of boats at present could be attributed to the reason that these boats have decayed during the long period. Thus, we need evidence of the absolute condition of Brim river in ancient times and of the existence of boats at that time to re-examine the claim.

Even if the two claims listed above are true, the conclusion that the so-called Palean baskets were not uniquely Palean is still problematic. At the very beginning, the passage asserts that woven baskets are found in the immediate vicinity of the prehistoric village of Palea. However, the definition of ‘immediate vicinity’ is ambiguous. Although from the modern perspective, these two villages are totally separated due to the Brim river between them, it’s still possible that, in ancient times, they were linked by bridge or shallow water beach. Based upon such an assumption, Palea and Lithos were not isolated and they can be integrated into a whole from the geographical perspective. If we want to reach this conclusion that woven baskets are not uniquely Palean, we will need more evidence to exclude the possibility that Lithos could be part of Palea in ancient time.

结尾:
To sum up, the evidence cited in the passage does not provide enough information to reach the conclusion that the so-called Palean baskets were not uniquely Palean. We need additional evidence, like the geographical condition of/between Palea and Lithos at their times, to better evaluate the argument.


作者: bigheiniu    时间: 2017-8-16 21:05:02

本帖最后由 bigheiniu 于 2017-8-18 23:52 编辑

Argument 53) The following appeared in a health magazine.

The citizens of Forsythe have adopted more healthful lifestyles. Their responses to a recent survey show that in their eating habits they conform more closely to government nutritional recommendations than they did ten years ago. Furthermore, there has been a fourfold increase in sales of food products containing kiran, a substance that a scientific study has shown reduces cholesterol. This trend is also evident in reduced sales of sulia, a food that few of the most healthy citizens regularly eat.

Write a response in which you discuss what specific evidence is needed to evaluate the argument and explain how the evidence would weaken or strengthen the argument.

claim:
The citizens of Forsythe have adopted more healthful lifestyles.
data:
a survey show that residents' eating habbits match the goverment nutritional recommendations more closely.
warrant:
1. similar to the recommnedations is the key for keeping health.
2. the survey's correctness
evidence:
1. more data about peopel's health in the past ten years.
2. more informatin about the recommendations.
3. whether the survey cover all kinds of people in the city.

data:
A fourfold increase in sales of food products containing kiran, a substance that a scientific study has shown reduces cholesterol.
warrant:
1. the food contains the kiran can reduce cholesterol.
2. individual consumption of kiran's food has been increasing.
3. the diseases only caught by high level of cholesterol
evidence:
1. more experimental results about whether the food containing kiran have the same effect on people's health with the labotary results.
2. data about whether individual consumption of the kiran's food have been increasing.
3. the data from hostital about whether the disease residents suffer are caused by cholesterol.

data:
This trend is also evident in reduced sales of sulia, a food that few of the most healthy citizens regularly eat.
warrant:
people eat less food containing sulia is healthier.
assumption:
1. individual consumption of sulia food have been decreasing.
2. the food containing sulia is harmful to health.


============正文分割线==============
开头
According to the author of the lecture, the citizens of Forsythe have adopted more healthful lifestyles given that people's eating habits more similar to government recommendations, consuming food rich in kiran and less in sulia. In order to evaluate the claim, we need to find more evidence about the recommendations, the influence of kiran and sulia food.

中间段
To begin with, we need some evidence about the recommendation's actual influence. The author merely points out that people's eating habits are more close to the recommendation. There is great possibility that although the food is health, people less likely to do sports. When they finish eating, they just go to sleep, or in their spare time, they stay behind the television instead of go for a walk. These spiritless life habits can also harm people's health even people have a proper diets. Moreover, with the development of technology, food's containing and the way they made has been changing year by year. So the static recommendations which has been last for ten years may not guide residents' eating properly, maybe with some outdated suggestions.

Secondly, we also need some evidence about the influence of the food's containing. The author says that people here prefer food consisting of more kiran rather than kiran. It is possible that the total consumption of these food production have increased, but the average sales for every individuals have decreased. Thus, the increase consumption of healthy food does not make any improvement on every single people. Besides, the food containing the kiran may not have the same effects on people's health compared to the results from the laboratory. With many other uncontrolled factors in people's daily life, the effect of kiran has been counterproductive maybe like air pollution or the nervous spirit among the residents. Without evidence that can eliminating other factors, we cannot say the increasing consumption of kiran's food can make people healthier.

Thirdly, we still need evidence that cholesterol is the drive factor of disease here. The author just correlate the less cholesterol with healthy lifestyle. Just like the speculation above, if people here like drinking or they are under the pressure from their work, food with less cholesterol does not stand that people here have a healthier lifestyle which actually omits many clandestine factors. So if the director of the government just rely on this, their optimistic policy about improving people's health will meet some trouble in the future.

结尾
In a nutshell, the author claim residents live in Forsythe have a healthier lifestyle. But actually, to stand for this claim, we need more evidence about the actual influence about the recommendation and food containing on people's daily life instead of from the assumption or the laboratory.

============正文分割线==============
中间段我是:
      1. 我还需要什么样证据.
      2. 作者在 magazine 提出的 data 是什么.
      3. 开始提出假设, 讨论其他可能情况
      4. 总结

评价41楼:
1. 问题都找出了, 针对河流展开了三种角度进行了攻击: a. 河流是否存在 b. 河流会存在, 是否很深. c. 河流很深, 船是否存在

2. 建议:
a. 关于开头引导语:
https://bbs.gter.net/thread-1795787-1-1.html
b. 这篇文章比较适合层层递进, 因为攻击点都是一环接一环, 不是一个并列的状态.
c. 那种层层递进的状态感觉是先说一个错误, 然后假设这个错误成立, 但是还是有其他的错误出现. 说的那篇 Argument 无论怎么样都是有问题的, 这个需要你自己按照轻重缓急排好序



作者: freefm    时间: 2017-8-16 22:06:46

Woven baskets characterized by a particular distinctive pattern have previously been found only in the immediate vicinity of the prehistoric village of Palea and therefore were believed to have been made only by the Palean people. Recently, however, archaeologists discovered such a "Palean" basket in Lithos, an ancient village across the Brim River from Palea. The Brim River is very deep and broad, and so the ancient Paleans could have crossed it only by boat, and no Palean boats have been found. Thus it follows that the so-called Palean baskets were not uniquely Palean.

Write a response in which you discuss what specific evidence is needed to evaluate the argument and explain how the evidence would weaken or strengthen the argument.

Claim: Since there is no boat found in the Palea village, Palean baskets were not unique.

Evidence required:1. The brim River was as deep and broad as it is.
                            2. Boat was the only way to cross the river
                            3.Wooden boat can last centeries.

Since there is no Palean boat being found in the Palean village, the author states that, regarding the fact that boating was the only way to go across the river, the Palean baskets were not unique in the area. The argument remians doubtful to me when several evidences are missing.

Evdence1

Evidence2

Evidence3

From my perspecive, the argument requires the evideces above to reach an cogent conclusion. If Evidence 1.2.3 supply sufficient support, thus it is sound to hold the point that Palean boats were not unique in the Palean village.
                  
作者: eerri2000    时间: 2017-8-16 22:22:14

本帖最后由 eerri2000 于 2017-8-16 22:36 编辑

151) Benton City residents have adopted healthier lifestyles. A recent survey of city residents shows that the eating habits of city residents conform more closely to government nutritional recommendations than they did ten years ago. During those ten years, local sales of food products containing kiran, a substance that a scientific study has shown reduces cholesterol, have increased fourfold, while sales of sulia, a food rarely eaten by the healthiest residents, have declined dramatically. Because of these positive changes in the eating habits of Benton City residents, we predict that the obesity rate in the city will soon be well below the national average.

Write a response in which you discuss what questions would need to be answered in order to decide whether the prediction and the argument on which it is based are reasonable. Be sure to explain how the answers to these questions would help to evaluate the prediction.


claim: Benton City residents have adopted healthier lifestyles.
data: eating habits of civilians conform more to government nutritional recommendations than they did ten years ago
warrant: the size of the sample of civilians investigated must be given
evidence:
1.not all of the civilians, especially those are under an unhealthy diet, would want to investigated;
2. the sample should be big enough and cover all the group including healthy residents and unhealthy residents.

claim: variance of the sales of food containing kiran and sulia indicates a healthier diet of residents
data: sales of kiran increase while sales of sulia decrease.
warrant: The changing of two food may not comprehensively reflect the shifting of diet style as a whole.
evidence:
1. Whether a diet of citizens is healthy could be judged only by taking into account the variance of all the food they consumed;
2.The changing of sales of all food sold in city should be displayed in this argument

claim: the obesity rate in the city will soon be well below the national average
data: Because of these positive changes in the eating habits of Benton City residents
warrant: Obesity rate in the city is solely related to the eating habits of citizens
evidence:
1. obesity rate is not exclusively depend on the diet conditions of residents of this city;
2. Sports habit of citizens also have a great influence on the obesity rate.


Intro:
In this passage, the author concluded that Benton City residents have adopted healthier lifestyles since more citizens conformed a healthy diet recommended by government today than ten years ago, which was supported by the increasing consume of the food containing kiran and the decreasing consume of the sulia. At first glance the arguer gives plausible reasons, however, if the author want to this argument acceptable, additional evidences about the sample size of citizens investigated and the variances of sales of all food consumed by residents should be given, the correlations of the diet conditions of residents and the obesity rate should also be future discussed. These aspect need to be considered critically to assess the strength of the argument.


Middle paragraph of TP1:
Firstly, an important assumption in this argument is that residents in Benton City have adopted healthier life styles because the citizens conformed more closely to the diet commended by government than they did ten years ago. This might be possible, but enough evidences were not given to prove this out. The size of the sample of civilians who are investigated must be given. If the sample only contain those people who are in healthy condition, the data showing the changing of diet of citizens would not be convincing. The sample should be big enough and cover all the group including both healthy and unhealthy residents.


写这一篇的时候,三个句子发现逻辑联系的很紧密,环环相扣。所以首段用了个which引导的定语从句,是想表明健康生活方式是因为遵循政府推荐的饮食,而这个被城市的food变化所证实。不知道这个句子写的对不对,还望指点。
句式逻辑方面还望指点

作者: powerfulbean    时间: 2017-8-16 23:22:02

1)Palean Basket unique

Woven baskets characterized by a particular distinctive pattern have previously been found only in the immediate vicinity of the prehistoric village of Palea and therefore were believed to have been made only by the Palean people. Recently, however, archaeologists discovered such a 'Palean' basket in Lithos, an ancient village across the Brim River from Palea. The Brim River is very deep and broad, and so the ancient Paleans could have crossed it only by boat, and no Palean boats have been found. Thus it follows that the so-called Palean baskets were not uniquely Palean.

Write a response in which you discuss what specific evidence is needed to evaluate the argument and explain how the evidence would weaken or strengthen the argument.

Background info: Woven baskets were once believed to have been made only by the Palean people because they have previously been found only in the immediate vicinity of the prehistoric village of Palea.

Claim : Palean people may be not the only one that made the basket.
Data: Archaeologists discovered such a 'Palean' basket in Lithos, an ancient village across the Brim River from Palea.
Warrant: Lithos located on the other side of the Palens, so the basket is made by people in lithos.
Assumption: Palean people could not bring the basket across the river.

Claim:It is difficult for Palean people to bring the basket across the river.
Data: The Brim River is very deep and broad.
Warrant : the river is deep and broad so it was not easy for them to cross the river.
Assumption: In the past, the river was as deep and broad as today.

Claim: Antient Palean people had no way to send the basket across the river.
Data:  and no Palean boats have been found.
Warrant : No Palean boats have been found which means that Palean people could not cross the river thus could not send the basket across the river.
Assumtion: they built the boat but were not preserved. They have no other way to carry the basket to the other side of the river.

Intro
According to the passage, it is concluded that the baskets which were previously only found in Palea were not only belong to Palean people. Citing the archaeologists’ discovery that same baskets were also found in Lithos and there is a wide river between them while no boat was found, the author believe it is  people in lithos produced these baskets themselves. Several assumptions have been made in the argument about the past condition of the river, the preservation of the antient boats and the methods to transport the baskets. These assumptions need ti be scrutinized critically to assess the strength of the argument.

Middle:
To begin with, it is assumed that the condition of the river in the area at the time such as width or depth were as same as today. While the river today is broad and deep, it does not mean the condition is the same in the past. There is possibility that in the past there were no river at this area and the geography or other disasters created it. People can easily traded commodities and baskets if there were no river. On the other hand, maybe the weather was extremely cold at the time and the river was frozen during the winter. Again the people from the two villages could simply walk across the river to trade.

作者: 很_简单    时间: 2017-8-16 23:34:35

Woven baskets characterized by a particular distinctive pattern have previously been found only in the immediate vicinity of the prehistoric village of Palea and therefore were believed to have been made only by the Palean people. Recently, however, archaeologists discovered such a 'Palean' basket in Lithos, an ancient village across the Brim River from Palea. The Brim River is very deep and broad, and so the ancient Paleans could have crossed it only by boat, and no Palean boats have been found. Thus it follows that the so-called Palean baskets were not uniquely Palean.

Write a response in which you discuss what specific evidence is needed to evaluate the argument and explain how the evidence would weaken or strengthen the argument.

C: so the ancient Paleans could have crossed it only by boat,
D: The Brim River is very deep and broad
W:

C; the so-called Palean baskets were not uniquely Palean.
D: no Palean boats have been found
W:it can not be made by Palean due to lack of vehicles.

In this argument, the author argues that the so-called Palean baskets which were considered only exist in the Palean area in the past can also be made in Lithos. To support his argument, the writer cites some evidence such as lacking vehicles and deep river to claim that these baskets can not be carried to Lithos by Palean. Although quite convincing the argument may seem at the first glance, more critical evidence is needed in order to full assess the writer’s conclusion.

To begin with, while the author provides some evidence concerning the river is very deep and broad to support his claim that the ancient Paleans could have crossed the river called The Brim only by boat, we can not hastily draw the conclusion that the river is as deep and broad as its ancient. We need more evidence to make sure that during the Paleans time, people could not pass the river due to its depth. If some evidence suggests that the river is not so deep that ancient Paleans do not have the ability and technology to pass it or even it is not existed in that time, the author’s conclusion will be greatly undermined. On the contrary, if the evidence supports the current it, then the conclusion may seem more conclusive.


To sum up, current information available in this argument does not sufficiently support the author’s conclusion that Palean baskets were not only existed in Palean area. We can not draw a conclusion solely based on the information he provided. As a result, we need more evidence to better evaluate the writer’s conclusion in the argument.

claim data 还比较好写,但是warrant不太理解什么意思,有大神帮忙解答吗?
作者: 游离雨    时间: 2017-8-17 10:04:25

tesolchina 发表于 2017-8-12 12:06
Arg 1 Palean Basket unique

Woven baskets characterized by a particular distinctive pattern have p ...

144) The citizens of Forsythe have adopted more healthful lifestyles. Their responses to a recent survey show that in their eating habits they conform more closely to government nutritional recommendations than they did ten years ago. Furthermore, there has been a fourfold increase in sales of food products containing kiran, a substance that a scientific study has shown reduces cholesterol. This trend is also evident in reduced sales of sulia, a food that few of the healthiest citizens regularly eat.

Write a response in which you examine the stated and/or unstated assumptions of the argument. Be sure to explain how the argument depends on these assumptions and what the implications are for the argument if the assumptions prove unwarranted.

In this argument, it is concluded that the citizens of Forsythe have adopted more healthful lifestyles than before. To support this, the author cites the results of a recent survey concerning people’s eathing habits, points out the increase of kiran selling and decline of sulia selling. However, a number of assumptions have been made in the argument about the interpretation of the survey results and the role the food products containning kiran or sulia play in people’s life.

To begin with, it is assumed that the responses to the survey could accurately reflect people’s eating habits and the results is representative of the condition of the general citizens. Actually, more questions need to be answers about the validity of the survey. On one hand, the respondents might not truly tell the answers about their eating habits for they would choose the one they think is the right thing but not the real. On the other hand, the respondents probably could not represent the general public in Forsythe and it is possible that they are mainly composed of young people who pay more attention to the health of lifestyle, while other people might remain the same as before. If the survey is not as valid as the author thought, the soundness of the argument might weaken and become less reliable.

Claim: The citizens of Forsythe have adopted more healthful lifestyles.
Data: Their responses to a recent survey
Warrant: show that in their eating habits they conform more closely to government nutritional recommendations

Claim: The citizens of Forsythe have adopted more healthful lifestyles.
Data: there has been a fourfold increase in sales of food products containing kiran
Warrant: a substance that a scientific study has shown reduces cholesterol.

Claim: The citizens of Forsythe have adopted more healthful lifestyles.
Data: reduced sales of sulia
Warrant: a food that few of the healthiest citizens regularly eat.
作者: wanghaitao19939    时间: 2017-8-17 10:20:35

53)The following appeared in a health magazine.

The citizens of Forsythe have adopted more healthful lifestyles. Their responses to a recent survey show that in their eating habits they conform more closely to government nutritional recommendations than they did ten years ago. Furthermore, there has been a fourfold increase in sales of food products containing kiran, a substance that a scientific study has shown reduces cholesterol. This trend is also evident in reduced sales of sulia, a food that few of the most healthy citizens regularly eat.

Write a response in which you discuss what specific evidence is needed to evaluate the argument and explain how the evidence would weaken or strengthen the argument.

Claim:
The citizens of Forsythe have adopted more healthful lifestyles.
Data:
1. a recent survey show that they conform more closely to government nutritional recommendations.
2. increase in sales of food products containing kiran that a scientific study has shown reduces cholesterol.
3. reduced sales of sulia, a food that few of the most healthy citizens regularly eat.
Warrant: the citizens adopt healthful lifestyles because they conform government nutritional recommendationsin a survey, buying more products containing kiran and don’t buy sulia.
Assumptions and Evidence:
1.few of the most healthy citizens eat sulia doesn’t mean that sulia is harm to healthful lifestyle, so reduced sales of sulia doesn’t mean a healthier lifestyle of people.
2.increase in sales of food products containing kiran doesn’t show a healthier lifestyle of people, on the contrary, it is possible that more people are suffering from over cholesterol, so they eat kiran to reduce it.
4.We need more evidence to compare the conformations to government nutritional recommendations 10years ago and now, there maybe no such a government nutritional recommendations 10years ago.
开头段:
The author of the health magazine asserts that the citizens of Forsythe have adopted more healthful lifestyles. To support this argument, the writer presents a response of a recent survey show and two changes in certain substance selling. Although the assumptions seem plausible, the argument will be seriously challenged if these evidence proved to be unwarranted.
中间段:
To start with, the citizens’ responses to a recent survey show that they conform more closely to government nutritional recommendations than they did ten years ago. Based on a unstated assumption that there has been such a government nutritional recommendations 10years ago, the writer endorse the citizens’ attitude toward nutrition has changed more healthful. However, the assumption is potentially problematic because there maybe no such a government nutritional recommendations 10years ago, even if we acknowledge this assumption, we need more evidence to compare the conformations to government nutritional recommendations 10years ago with now. If this is the case, the writer’s assumption will not hold and the conclusion will be clearly weakened.

Additionally, by stating that increase in sales of food products containing kiran represent a healthier lifestyle of people, the writer rules out the opposite situation. We need to re-examine the assumption that selling more kiran food is the symbol of a healthful lifestyle. It is likely on the contrary, more people are suffering from over cholesterol, so they eat kiran to reduce it rather than as the writer’s assumption. In addition,the results of medical examination will tell us more about the real health condition of citizens in Forsythe. If the worse helth condition is true, then we can not take increase in sales of food products containing kiran as a healthier lifestyle of people.

What’s more, the writer demonstrate that reduced sales of sulia, a food that few of the most healthy citizens regularly eat, means people lead a more healthful lifestyle. Whether or not the sulia should be discriminated to be a unhealthful substance is questionable. For example, if the sulia is innocuous to health, it’s reduced selling should not be a standard of health lifestyle or not. More over, the reduction of sulia selling may be caused by a reduction of its production. For that reason, the reduced sales of sulia may not be a necessary key to lifestyle change. As a result, the argument is debatable.
加了一些连接词和看起来很有逻辑的过渡,我觉得比昨天的好多了,看来写得好就是让自己写的看起来好。
作者: youzi42    时间: 2017-8-17 14:38:47

Arg 1 Palean Basket unique

Woven baskets characterized by a particular distinctive pattern have previously been found only in the immediate vicinity of the prehistoric village of Palea and therefore were believed to have been made only by the Palean people. Recently, however, archaeologists discovered such a 'Palean' basket in Lithos, an ancient village across the Brim River from Palea. The Brim River is very deep and broad, and so the ancient Paleans could have crossed it only by boat, and no Palean boats have been found. Thus it follows that the so-called Palean baskets were not uniquely Palean.

Write a response in which you discuss what specific evidence is needed to evaluate the argument and explain how the evidence would weaken or strengthen the argument.

Claim : Woven baskets characterized by a particular distinctive pattern have been made only by the Palean people.
Data :Woven baskets found only in the immediate vicinity of the prehistoric village of Palea
Warrent : The woven baskets only appears in the Palea
Evidence :
- The people who used to live in here , seldom has trade with other people from other villages .
- The woven baskets is so special that it can only made in this villages for some reasons .
- The people made this woven baskets originally .

Claim : the so-called Palean baskets were not uniquely Palean.
Data  :archaeologists discovered such a 'Palean' basket in Lithos, an ancient village across the Brim River from Palea.ee
Warrent :The Brim River is very deep and broad, and so the ancient Paleans could have crossed it only by boat, and no Palean boats have been found.
Evidence :
- In the history of the ancient village , there was no contact with the people from the prehistoric villages.
- The river exsists in the ancient time too , and as deep and broad as now.


  The argument is about the origin of woven baskets , characterized by a particular distinctive pattern : the woven baskets were  first found in the immediate vicinity of the prehistoric village of Palea , however, an archaeologists' discovery found that such a 'Palean' basket in Lithos, an ancient village across the Brim River from Palea , and no evidence shows there used to have Plean .
Whether the so-called Palean baskets were uniquely Palean or not remains question , it lies in the evidences that show the possibilities of contact between the two villages . Thus , before making a judgement , we still need some evidence : the river exists since the prehistory and is as deep and broad as now , there didn't have a bridge before  , and the evidence that whether the resources of making woven baskets is special certain areas or not  .

  According to the argument , there is  the Brim River between the two villages , the ancient Paleans could have crossed it only by boat . However , we can't come to a conclusion given we still need evidence , that is,  the Brim River exists since or before the Palea's settement , what's more , the river should be as broad and deep as now so the Palea can only cross it by the boat . If there were not a river , or the river drys at some times , hence , it's no wonder that there isn't any boat be found then , moreover ,  there is possibility that the two villages may have trade in this basket , leaving the origin of the woven basket a mystery .



作者: ChiuChiKwan    时间: 2017-8-17 16:34:08

53)The following appeared in a health magazine.

The citizens of Forsythe have adopted more healthful lifestyles. Their responses to a recent survey show that in their eating habits they conform more closely to government nutritional recommendations than they did ten years ago. Furthermore, there has been a fourfold increase in sales of food products containing kiran, a substance that a scientific study has shown reduces cholesterol. This trend is also evident in reduced sales of sulia, a food that few of the most healthy citizens regularly eat.

Write a response in which you discuss what specific evidence is needed to evaluate the argument and explain how the evidence would weaken or strengthen the argument.


Claim: The evidence of the reliability of the responses is required.
Data: Their responses to a recent survey show that in their eating habits they conform more closely to government nutritional recommendations than they did ten years ago.
Warrant:Whether the responses to the recent survey is real.
What if people lie to the survey to leave a good impression?


Claim: There is no evidence that eating habits of citizens are improved by a fourfold increase in food products containing kiran.
Data: There has been a fourfold increase in sales of food products containing kiran and scientific study has shown it reduces cholesterol.
Warrant:
1.There is no evidence that the increase in food products containing Kiran doesn’t mean local people will eat them. Chances are that the most of consumers are from other areas. The expansion  of this kind of food may bring more profits.
2. Since the fourfold increase, some evidences are required that there is no harm if the consumption of this kind of food is excessive.




Claim: The reduction of sulia doesn’t mean citizens begin to eat healthy food.
Data:This trend is also evident in reduced sales of sulia, a food that few of the most healthy citizens regularly eat.
Warrant: They may choose another kind of unhealthy food to eat.

Claim:Even the citizens of Forsythe start to have a healthier diet, no evidence shows that the citizens of Forsythe have adopted more healthful lifestyles in sports, recreation, sleep and other ways.
Data:Above all.
Warrant:Healthy lifestyles contains many aspects, such as diets, sports, sleep and so on. The evidence that there is a tendency to do sports in this city or other is failed to provide.


Introductory paragraph:
  The health magazine announces that life styles of citizens of Forsythe are better than before by citing the responses to the recent survey, the increasing purchase of food with Kiran and the deduction of sales of sulia. However, these phenomenon couldn’t reach a conclusion that the citizens of Forsythe have adopted more healthful lifestyles. Several evidences are fail to provide.

Middle paragraph:TS3
  It is assumed that the citizens have healthier lifestyles owing to reduced sales of sulia, a food that few of the most healthy citizens regularly eat. However, the editors of this magazine fail to provide evidence whether people change their taste and prefer other kinds of unhealthy food. What’s more, chances are that the reason why sale of sulia deduces is that exports of sulia is curtailed. Thus it’s still remains an open question whether this reduction really means people start to choose healthy diets instead of unhealthy one.
作者: woaka    时间: 2017-8-17 16:50:07

本帖最后由 woaka 于 2017-8-17 17:11 编辑

144)  The citizens of Forsythe have adopted more healthful lifestyles. Their responses to a recent survey show that in their eating habits they conform more closely to government nutritional recommendations than they did ten years ago. Furthermore, there has been a fourfold increase in sales of food products containing kiran, a substance that a scientific study has shown reduces cholesterol. This trend is also evident in reduced sales of sulia, a food that few of the healthiest citizens regularly eat.

Write a response in which you examine the stated and/or unstated assumptions of the argument. Be sure to explain how the argument depends on these assumptions and what the implications are for the argument if the assumptions prove unwarranted.

Claim: The citizens of Forsythe have adopted more healthful lifestyles.
Data: Their responses to a recent survey show that in their eating habits they conform more closely to government nutritional recommendations than they did ten years ago.
Warrant: (having an eating habit which is close to the government recommendation means a healthier lifestyle) implied
Assumptions:
1)        The survey is persuasive and objective
2)        The survey did ten years ago is credible
3)        The eating habit which is close to the government recommendation is healthier

Claim
: The citizens of Forsythe have adopted more healthful lifestyles.
Data: there has been a fourfold increase in sales of food products containing kiran, a substance that a scientific study has shown reduces cholesterol
Warrant: (the increase in the sales of food products containing kiran in Forsythe indicates the healthful lifestyles of citizens) implied
Assumptions:
1)        It is people’s concern about healthful lifestyles that leads the increasing sales of healthful food products, such as the one containing kiran
2)        The increase in the sales of food products containing kiran is significant enough in amount and benefited many of the citizens
3)        The reduction of cholesterol means healthful

Claim: The citizens of Forsythe have adopted more healthful lifestyles.
Data: the reduced sales of sulia, a food that few of the healthiest citizens regularly eat.
Warrant: The decline in sales of suila has proven the trend of advocating healthy lifestyles in Forsythe
Assumption:
1)        The foods that few of the healthiest citizens regularly eat are unhealthful
2)        The decline in sales of suila shows people's attention to health

Introductory paragraph
The author argues that the citizens of Forsythe have adopted more healthful lifestyles. A number of assumptions have been made about the recent survey concerning eating habits of citizens, and the sales records of two types of food. We need to examine these assumptions carefully to decide if the argument is reasonable.

Middle paragraph
To begin with, it is assumed that the surveys about citizens’ eating habits could draw the conclusion that the city residents adopted more healthful lifestyles than they did ten years ago. It’s not clear, however, the scope and validity of that survey. For one thing, could these sampled respondents truly represent the Forsythe citizens? If the respondents were mainly from a particular socioeconomic class or the amount of respondents was not big enough, for example, the survey results could not be generalized to the entire population in Forsythe. For another thing, is the eating habit which is close to the government recommendation healthier? As we all known, the standard for a healthful lifestyle is not unique. Even if this survey could prove that people's eating habits are closer to the government nutritional recommendations, the author cannot say that people's lifestyles are healthier without scientific research. Unless the survey is fully representative, valid, and reliable, it cannot be used to effectively back the authors’ argument.


5楼同学的习作点评
1. 您的开头段写的不错,很有条理,讲明了作者的的几个主要论据,然后依次列出四个需要被回答的问题,便于组织文章的结构,我要向你学习。但是陈述作者结论的时候第一句和第二句的后半句显得就有点重复了,可以把一二句组合在一起,作为对全文逻辑结构的简单复述。
2. 中间段
1) 语言显得比第一段弱了一点(比如draw to a conclusion,应该没有to吧)
2) 好像在哪里看到过用cannot 比can’t正式一些(仅供参考)
3) 倒数第二句应该是对政府推荐的健康性以及现在的饮食习惯更健康了做出让步(承认),所以这里的less应该有问题吧…
4) 这一段的论述比我的文章内容要丰富的多,感觉自己废话太多了。向你学习


作者: 丽丽酱香饼    时间: 2017-8-17 16:55:21

本帖最后由 丽丽酱香饼 于 2017-8-17 16:59 编辑

ARG 1. Palean Basket unique

Woven baskets characterized by a particular distinctive pattern have previously been found only in the immediate vicinity of the prehistoric village of Palea and therefore were believed to have been made only by the Palean people. Recently, however, archaeologists discovered such a 'Palean' basket in Lithos, an ancient village across the Brim River from Palea. The Brim River is very deep and broad, and so the ancient Paleans could have crossed it only by boat, and no Palean boats have been found. Thus it follows that the so-called Palean baskets were not uniquely Palean.

Write a response in which you discuss what specific evidence is needed to evaluate the argument and explain how the evidence would weaken or strengthen the argument.

Claim: ancient Paleans could have crossed it only by boat
Data: Archaeologists discovered such a 'Palean' basket in Lithos, an ancient village across the Brim River from Palea. The Brim River is very deep and broad.
Warrant: Because the Brim River is very deep and broad, so ancient Paleans could only crossed it by boat.

Evidence needed :
1.        information about other possible ways for basket to cross the river
a.        Bridge?
b.        Frozen in winter?
c.        Carried just by water?
2.        geographic Information about the river in ancient time
a.        very deep and broad as today?
b.        Some parts connected?

Claim: Palean baskets could not be brought to Lithos
Data: no Palean boats have been found
Warrant: boat is the only way to pass the basket

Evidence needed:
1.        possible boat materials
2.        the possibility of all boats are been damaged

Claim: the so-called Palean baskets were not uniquely Palean
Data: the basket was found in Lithos and there is no boat founded
Warrant: because the basket was found in Lithos, Lithos could make the basket

Evidence needed:
1.        the material of the basket to prove Lithos could make the basket
2.        information to exclude the possibility of trade of the basket from other place

Introductory paragraph

In this argument, the author states that the Palean baskets were not unique to be made by Palean because they have been found in Lithos across the river. More evidence is needed to evaluate this argument: geographical information of this area, the possible technology of boat and bridge, and the characteristics of the baskets.

Middle paragraph

A important piece of evidence needed is the information of geography at the time in this area. Specifically, we want to know whether the river was deep and broad and if there were some land parts of the two village connected together. It is possible that the geographical characteristics at the time were so different from today. If the river was shallow and narrow, people in Palean and Lithos could walk to each other, and the baskets could be traded across the river. Maybe in somewhere of the river, two village were connected by some land parts which made people easy to walk through. Again they could trade the baskets across the river. So, more information of geography is needed to assess how likely these scenario are.

评6楼:
1.        Evidence不可数名词:
[U] facts or signs that show clearly that something exists or is true
证据,证明
2.        首段最好开门见山给出thesis statement,说明缺了哪些证据
3.        It’s so cold——maybe it was

作者: Joker—XYZ    时间: 2017-8-17 17:00:06

53) The following appeared in a health magazine.

The citizens of Forsythe have adopted more healthful lifestyles. Their responses to a recent survey show that in their eating habits they conform more closely to government nutritional recommendations than they did ten years ago. Furthermore, there has been a fourfold increase in sales of food products containing kiran, a substance that a scientific study has shown reduces cholesterol. This trend is also evident in reduced sales of sulia, a food that few of the most healthy citizens regularly eat.

Write a response in which you discuss what specific evidence is needed to evaluate the argument and explain how the evidence would weaken or strengthen the argument

Claim: The citizens of Forsythe have adopted more healthful lifestyles.
Data: In their eating habits they conform more closely to government nutritional recommendations than they did ten years ago.
Warrant:Good eating habits due to more healthful lifestyle.

Questions:How did them do the survey on citizens eating habits?
                      Did this survey cover the major part of citizens?
Evidence:Survey methods and the range of participants.
Assumption:The conclusion is true and forceful.

Claim:The citizens of Forsythe have adopted more healthful lifestyles.
Data: There has been a fourfold increase in sales of food products containing kiran, a substance that a scientific study has shown reduces cholesterol.
Warrant:The increase in sales of food products containing kiran due to health eating habits.

Questions:Are there any other factors influence the sales of food products containing kiran.
Evidence:The price changes of food products containing kiran.
Assumption:People eat more food products containing kiran to keep healthy .

Claim:The citizens of Forsythe have adopted more healthful lifestyles.
Data: This trend is also evident in reduced sales of sulia, a food that few of the most healthy citizens regularly eat.
Warrant:Good eating habits due to more healthful lifestyle.

Questions:Is there obviously reduce on sulia output?
Evidence: Output data of sulia.
Assumption:People realize sulia's harmful influence on health.

Introductory paragraph:

According to the director in a health magazine,the citizens of Forsythe have adopted more healthful lifestyles because of a better eating habit. Citing the changes on sales of different kinds of food, the director argues that the citizens' eating habits they conform more closely to government nutritional recommendations than they did ten years ago. A number of assumptions have been made in the argument about the survey results, the healthful lifestyles. These assumptions need to be scrutinized critically to assess the strength of the argument.

Middle paragraph:1
To begin with, it is assumed that the survey result accurately reflects the lifestyles of citizens in Forsythe.  While the way of the survey and its range of participants are unknown, may the participants were concentrated in an area or age range. Seniors always pay more attention to their health and there would be less young people to do so. If the survey did not contain a large range of ages, the result may not be helpful.
作者: grepph    时间: 2017-8-17 17:05:23

53) The following appeared in a health magazine.

The citizens of Forsythe have adopted more healthful lifestyles. Their responses to a recent survey show that in their eating habits they conform more closely to government nutritional recommendations than they did ten years ago. Furthermore, there has been a fourfold increase in sales of food products containing kiran, a substance that a scientific study has shown reduces cholesterol. This trend is also evident in reduced sales of sulia, a food that few of the most healthy citizens regularly eat.

Write a response in which you discuss what specific evidence is needed to evaluate the argument and explain how the evidence would weaken or strengthen the argument.

Intro paragraph:
In the health magazine, it is said that the citizens of Forsythe have been used to healthy lifestyles, according to the survey of their eating habits, the increase in sales of food products containing kiran and the decrease in the sales of sulia. However, there is still some evidence needed to evaluate the argument about the survey result, the sales of food products containing kiran and the sales of sulia.

Claim: The citizens of Forsythe have adopted more healthful lifestyles.
Data:Their responses to a recent survey show that in their eating habits they conform more closely to government nutritional recommendations than they did ten years ago.
Warrant: People are more willing to follow the government nutritional recommendations.
Evidence needed: Prove for the reliability of the survey (the number of people being surveyed, the quality of the questions in the survey, the realness of the answers).

Middle paragraph:
Firstly, although the survey shows that people’s eating habits tend to be more closely related to government nutritional recommendations, evidence is needed to prove the reliability of the survey. Maybe the ratio of people in the survey who is used to healthy lifestyles is larger than that of people ten years ago, while the ratio of people with healthy lifestyles in the whole city is few than that of people ten years ago. Also, the questions in the survey maybe not good enough to make people in the survey truly express their opinions. If this is true, then the survey can’t represent the habits of the whole city.

Claim: The citizens of Forsythe have adopted more healthful lifestyles.
Data: There has been a fourfold increase in sales of food products containing kiran, a substance that a scientific study has shown reduces cholesterol.
Warrant: people choose to buy more healthy food.
Evidence needed: Information about other substances in the food; The sales of other food products having the same effect but containing no kiran.

Claim: The citizens of Forsythe have adopted more healthful lifestyles.
Data: This trend is also evident in reduced sales of sulia, a food that few of the most healthy citizens regularly eat.
Warrant: People choose to buy few unhealthy food.
Evidence needed: information about the quality of sulia now and before; sales of other food products that has the same effect as sulia.
作者: 东扯扯    时间: 2017-8-17 19:15:44

kyika 发表于 2017-8-16 14:26
151) Benton City residents have adopted healthier lifestyles. A recent survey of city residents show ...

今天才注意到还有点评。因为昨天的第一组argument我看大家都没点评,就以为argument不点评。
首先,多谢指正和建议。
关于第一个C-D-W,我是在做Toulmin's cycle时按Simon Wang的指导,尽可能把题目中的拆解出一个个C-D-W cycle,前天的第一组中我拆的特别细,感觉很零碎,所以这一次是根据题目中的final conclusion来列C-D-W,即与conclusion相关的我都列出来。但如你所说,其实我写的第一个C-D-W也感觉更多的是背景,与conclusion的联系并没有那么紧密和直接,所以我真正写全文的时候,会尽量挑更trenchant的点来argue(即多拓展我写的后两个C-D-W)。
我个人认为It is assumed that问题不大,因为我是先抛出statement里存在的问题从而引出还欠缺的evidence。当然你说的那样的确是更加简洁直接的回应题目要求,是对的。
谢谢耐心点评。

作者: sweepyou    时间: 2017-8-17 19:53:00

本帖最后由 sweepyou 于 2017-8-17 20:09 编辑

1)
Woven baskets characterized by a particular distinctive pattern have previously been found only in the immediate vicinity of the prehistoric village of Palea and therefore were believed to have been made only by the Palean people. Recently, however, archaeologists discovered such a 'Palean' basket in Lithos, an ancient village across the Brim River from Palea. The Brim River is very deep and broad, and so the ancient Paleans could have crossed it only by boat, and no Palean boats have been found. Thus it follows that the so-called Palean baskets were not uniquely Palean.

Write a response in which you discuss what specific evidence is needed to evaluate the argument and explain how the evidence would weaken or strengthen the argument.

Claim: The Palean baskets could travel across the river only by boat.
Data: The Brim River is very deep and broad now.
Warrant: The ancient Brim River was as deep and wide as it is now, so it was impossible to build a bridge. There were no other means for the baskets to travel across the river if it was deep and wide.
Evidence1: The geography condition of the ancient Brim River.
Evidence2:Whether the river froze during the winter.
Evidence3:The material of the Palean baskets.

Claim: There was no boat to cross over the ancient Brim River.
Data: no Palean boats have been found.
Warrant: If no Palean boat exists, it was impossible for the ancient people to cross the river by boat.
Evidence1: Whether boats from Lithos existed.
Evidence2: The material of the boat.
Evidence3: Geographical condition in the local area (factors that might took the boat remains far from the local area).

According to the author, the Palean baskets were not unique to Palean, based on the fact that a 'Palean' basket was found in an adjacent village, and that there were seemingly no means for the basket to travel from Plea village to the adjacent one. However, in order to better evaluate the argument, we need more evidence about the geographical condition as well as the climate of the Brim River area in the ancient times, the materials the ancient people use to build a boat, and the material of the Palean basket.

The author claims that it was impossible for the basket to travel across the Brim River without a boat, because of the depth and width it has now. However, we need more evidence about the geographical condition about the ancient Brim River. It is possible that the river was shallow enough in the ancient times for the people to wade, or that the river was narrow enough to build a bridge, so the ancient villagers could take the basket to the adjacent village. Besides, we also need the local climate data. If it was cold enough for the Brim River to freeze in the winter, people can walk across the area, despite the depth and width of the river. What's more, we need to examine the material of the Palean basket. If the basket was made of wood or bamboos, which made the basket light enough to float on water, the river might carry it to Lithos.


点评:13楼
开头段:个人觉得firstly后面的这句是可以不用写的,因为这属于背景介绍一类,这里面并没有需要提供证据(也就是可驳斥)的点。

第一段没太看明白,个人觉得需要讨论的就是篮子有没有可能是从Palea而来的。不明白你这里讨论材料是针对什么?

In addition这段挑不出毛病,而且比较简洁。不过记得老师说过,新AW不要出现flaw这类词,因为作者的观点未必就是错的,只是需要更多证据。

Even if这段,同理,不要出现problematic。
个人觉得这段和上段有点重复了,你是想说明由于巨大的地质变化,这两个村子实际上在古时候很近吗?这和上一段的river was shallow or narrow实际是一个意思,都是说可以架桥或者涉水的意思吧?

困惑的地方,请老师或者其他小伙伴回答:
关于arg1的审题:如果这种篮子的编织技术是P村教给L村的,那么这还能算是P篮子吗?是不是此篮子已经不算unique了呢?

另外,我之前练习argu一直是3+1法,似乎和现在这种方法不太一样,Toulmin必须要写出warrant,不过我好像更习惯分层然后直接写出待解决的assumption/question/evidence。
作者: GoldenString    时间: 2017-8-17 20:43:12

Arg 1 Palean Basket unique

Woven baskets characterized by a particular distinctive pattern have previously been found only in the immediate vicinity of the prehistoric village of Palea and therefore were believed to have been made only by the Palean people. Recently, however, archaeologists discovered such a 'Palean' basket in Lithos, an ancient village across the Brim River from Palea. The Brim River is very deep and broad, and so the ancient Paleans could have crossed it only by boat, and no Palean boats have been found. Thus it follows that the so-called Palean baskets were not uniquely Palean.

Write a response in which you discuss what specific evidence is needed to evaluate the argument and explain how the evidence would weaken or strengthen the argument.


Claim: the so-called Palean baskets were not uniquely Palean.
Data:a 'Palean' basket was found in Lithos.
  /   The Brim River is very deep and broad, and so the ancient Paleans could have crossed it only by boat,
/and no Palean boats have been found.
Warrant: Because no Palean boats have been found, Palean people can't reach Litho without boats. It can believed that the 'Palean' basket in Lithos discovered by archaeologists was made by Lithos people.

开头段:According to the argument, the so-called Palean baskets maybe had been made by other people not only Palean people as the same baskets have been found in Lithos, where is an adjacent village of Palea ,which is separated from Lithos by deep and broad Brim River.Given that the ancient Paleans could have crossed it only by boat, and no Palean boats have been found,  It's believed that the baskets were not uniquely Palean. In order to evaluate this argument, we need more evidence about geographic and climate information of the areas ,the technologis of building boats and bridges , the material of the baskets and the environmentalbackgrounds of the areas.

中间段:Although the author believes that only by boats which were still not found  could Palean people carry the baskets from Palea to Lethos , we can't deny that maybe there are other reasonable scenarios. First of all, the Brim River maybe at the time was not as deep and broad as it is now. If  Brim River was so shallow and narrow that people on both sides could deal with baskets by walking across the river. For another, it is not of impossiblity that maybe there was no Brim River between the two villagea at the time. What's more, if the climate is so cold  that the river was frozen in winter, people on both sides could also trade baskets by walking across the river without the needs of boats. Therefore, we need more information about geographic and climate of the areas to assess the possibility of the above scenarios.


点评20楼:
开头段有点太啰嗦了,应该概括总结一下,一些细节点东西可以省略,比如Woven baskets characterized by a particular distinctive pattern可以概括为the baskets.
中间段不错,有理有据,很充实!
作者: liwenhao19    时间: 2017-8-17 20:58:54

tesolchina 发表于 2017-8-12 12:06
Arg 1 Palean Basket unique

Woven baskets characterized by a particular distinctive pattern have p ...

Arg 1 Palean Basket unique

Woven baskets characterized by a particular distinctive pattern have previously been found only in the immediate vicinity of the prehistoric village of Palea and therefore were believed to have been made only by the Palean people. Recently, however, archaeologists discovered such a 'Palean' basket in Lithos, an ancient village across the Brim River from Palea. The Brim River is very deep and broad, and so the ancient Paleans could have crossed it only by boat, and no Palean boats have been found. Thus it follows that the so-called Palean baskets were not uniquely Palean.

Write a response in which you discuss what specific evidence is needed to evaluate the argument and explain how the evidence would weaken or strengthen the argument.

Claim: Palean people are not the only one who can make the so called palean baskets.
Data: palean baskets were found in Lithos(L), where is separated from Palea(P) by a deep and broad river, and no palean boats have been found.
Warrant: the geography did not change over time, and the baskets can only cross the river by Palean boats, and if there were Palean boats before, we must be able to find them today.

Intro:
The author claim that Lithos people can also make the so-called palean baskets since those baskets were also found in Lithos, which is separated from Palean by a deep and broad river and no Palean boats were found. However, we need more evidence about the geography condition of this area at that time and the technologies of people living in those areas at that time in order to decide whether the facts the author listed is enough to support his claim.

Body 1:
Firstly, the author argues that those baskets could not come from P, since there is deep and broad river between P and L. However, we need more evidence about the geography condition of this area at the time to evaluate the argument, because geography conditions change over time, the river is deep and broad today does not guarantee that it was so during that time. May be,  at the time the river was shallow and narrow enough that people could swim or walk across the river. If those were the cases, the authors’ argument is greatly weakened.

作者: HelloC424    时间: 2017-8-17 21:15:05

53) The following appeared in a health magazine.

The citizens of Forsythe have adopted more healthful lifestyles. Their responses to a recent survey show that in their eating habits they conform more closely to government nutritional recommendations than they did ten years ago. Furthermore, there has been a fourfold increase in sales of food products containing kiran, a substance that a scientific study has shown reduces cholesterol. This trend is also evident in reduced sales of sulia, a food that few of the most healthy citizens regularly eat.

Write a response in which you discuss what specific evidence is needed to evaluate the argument and explain how the evidence would weaken or strengthen the argument.



开头段:

A magazine did a survey through citizens of Forsythe revealed that they have adopted a kind of more healthful lifestyles due to three aspects: more consistent eating habits to the 10-year-old recommended nutritional standards, increasing sales in a more healthful food products and decreasing sales of a food that seldom eaten by healthy citizens.
From my perspective that we should scrutinize critically to this argument and need more information.
中间段:
First, more information about the survey among citizens should be offered. The argument did not showed the results of whether  the nutritional recommendations may have changed in 10 years. Did the researcher compare different results between these 10 years? Because with the development of our science, there might be some changes of what we thought is correct in the past. If this standards changed, the results in the above may not be valid enough to support the claim.

结尾段:
In conclusion, evaluating wether the citizens of Forsythe are more healthful through referring to the government recommended standards may be helpful but we should be attention to the date because conditions and environment are changed fleetingly. Also, observing the eating habits from any specific kind of food may also be helpful but may not lead to a valid and concrete result. To gain the claim of the argument, we need more datas from scientific survey.


claim:The citizens of Forsythe have adopted more healthful lifestyles.
data1: The citizens adopted eating habits that are more consist to the nutritional recommendation from government 10 years ago.
warrant: The nutritional standards of government was official and healthy.

question:  Are there any scientific datas show that the government standards are still healthful? In ten years, are there any changes of the standard? Because with the development of our science, there might be some changes of what we thought is correct in the past.
evidence: Scientific proof shows that the nutritional  recommendations are still efficacious.
assumption: That their consistency of the government recommended food standards  still remain powerful and healthful.

data2: Sales of food that has been proved could reduced cholesterol are increasing.
warrant: Kiran is healthy. Increasing intake of this healthy food that contains kiran will help the citizen be more healthful.

question: How much do a person intake Kiran will be healthful? We need more specific datas. Besides, what if those food which contains kiran also have other elements such as sugar and fat that are harmful to health?
evidence: Need scientific data to inform us about the dose of this kind of food and other detail about that this food is free of other unhealthy elements.
assumption: These food that contains kiran is healthy for citizens.

data3:Sales of food that healthy citizens will not regularly eat are decreasing.
warrant: Decreasing to eat the foods are regarded as unhealthy is benefit for the citizens’ health.

questions: Are there any beneficial elements are also contained in this kind of food. Need more specific data to show this kind of food is unhealthy for people. Maybe this result is only ostensible and only due to the preferences of people who are regarded as healthy.
evidence: Scientific results of whether this kind of food in harmful for people’s health.
assumption: Decreasing the intake of this unhealthy food.


对照了老师的范文,两个问题:
1.概括不够精准
2.老师写人们饮食习惯变了,我写政府标准变了。人们饮食习惯变了更好一点。
作者: 国产耗子    时间: 2017-8-17 21:16:05

144) The citizens of Forsythe have adopted more healthful lifestyles. Their responses to a recent survey show that in their eating habits they conform more closely to government nutritional recommendations than they did ten years ago. Furthermore, there has been a fourfold increase in sales of food products containing kiran, a substance that a scientific study has shown reduces cholesterol. This trend is also evident in reduced sales of sulia, a food that few of the healthiest citizens regularly eat.

Write a response in which you examine the stated and/or unstated assumptions of the argument. Be sure to explain how the argument depends on these assumptions and what the implications are for the argument if the assumptions prove unwarranted.

Claim: The citizens of Forsythe have adopted more healthful lifestyles.

Data: Their responses to a recent survey show that in their eating habits they conform more closely to government nutritional recommendations than they did ten years ago.

Warrant: the survey can represent most of the citizen and is the really eating habit.

Assumption:The recommendation from government is can make people live healthier than others. -there are not other eating habits they can follow.

Claim: The citizens of Forsythe have adopted more healthful lifestyles.

Data: there has been a fourfold increase in sales of food products containing kiran, a substance that a scientific study has shown reduces cholesterol.

Warrant: eating food products containing kiran cam make people healthier.
Assumption: There are not other food products which can reduce cholesterol, too. The customer who eating Kiran are health people.

Claim:The citizens of Forsythe have adopted more healthful lifestyles.

Data:This trend is also evident in reduced sales of sulia, a food that few of the healthiest citizens regularly eat.

Warrant: sulia is one kind of food can’t make people healthy.

Assumption: people who don’t want to buy sulia because sulia is not a healthy food.

开头:According to this article, the writer claim that the citizens of Forthythe are living in more healthful lifestyle. The writer citing the survey, the increasing sale of food with Kiran, and the decreasing sale of sulia to prove his/her view. However, assumptions have to be adjusted to make this article more strength.

中间段
First of all, it is assumed that the recommendation from government is can make people live healthier than other recommendation. The survey mentioned that citizens conform more closely to government recommendation. Maybe, 10 years ago, citizens have their own eating habits which are way more better the government’s advice today. If in this situation, then we can’t say people in this city today are living more healthier than 10 years go.
作者: Morfire    时间: 2017-8-17 21:26:41

本帖最后由 Morfire 于 2017-8-17 22:04 编辑

151) Benton City residents have adopted healthier lifestyles. A recent survey of city residents shows that the eating habits of city residents conform more closely to government nutritional recommendations than they did ten years ago. During those ten years, local sales of food products containing kiran, a substance that a scientific study has shown reduces cholesterol, have increased fourfold, while sales of sulia, a food rarely eaten by the healthiest residents, have declined dramatically. Because of these positive changes in the eating habits of Benton City residents, we predict that the obesity rate in the city will soon be well below the national average.

Write a response in which you discuss what questions would need to be answered in order to decide whether the prediction and the argument on which it is based are reasonable. Be sure to explain how the answers to these questions would help to evaluate the prediction.

1
Claim: Benton City residents have adopted healthier lifestyles
Data: a survey
Warrant: it is convincing

2
Claim: eat healthier
data: local sales of food products containing kiran, a substance that a scientific study has shown reduces cholesterol, have increased fourfold, while sales of sulia, a food rarely eaten by the healthiest residents, have declined dramatically
Warrant: it can reflect accurately lifestyle

3
Claim: the obesity rate in the city will soon be well below the national average
Data: positive changes in the eating habits
Warrant: eating habits help a lot

While it may be forthcoming that people in Benton City have an affinity for healthy lifestyle, and the obesity rate will be lower than the national average, this argument dose not necessarily prove its conclusion, and needs more information to make it sound in terms of the survey, the food, and eating habits.
段1:survey样本问题(抽样数量、随机性),权威与否。可能有些地方的人更健康。
段2:even if 段1,人们可能出于食物口味变化、商场供货减少、价格变化等其他原因改变购物风格。其他食物也需要证据。
段3:
Even if people choose the food that can reduce cholesterol on the purpose of eating healthily, it still based on the assumption that positive changes of diet could obviously and profoundly get residents in Benton City rid of obesity. Although, it is unassailable that eating habits contribute to the overall health, we can not be sure about the extend. We do not know if people there eat healthier but in the same time do less exercise, which, in some cases, might even increase the rate of obesity. Hence, this argument should provide the sports data while talking about food in order to convince readers that the obesity rate will soon be lower than the national average.

评:30楼
开头段:接连两个at that time,删掉前面那个吧?
whether… are…

主体:
Geographical transformation from that time也可以用来替换
May be,   是Maybe,
May be,  at the time the river was shallow and narrow enough that people could swim or walk across the river.我觉得这边后面可以在说清楚一点:thus, P could bring their baskets to other places through the river.


作者: TaylorW    时间: 2017-8-17 21:29:07

53) The following appeared in a health magazine.

The citizens of Forsythe have adopted more healthful lifestyles. Their responses to a recent survey show that in their eating habits they conform more closely to government nutritional recommendations than they did ten years ago. Furthermore, there has been a fourfold increase in sales of food products containing kiran, a substance that a scientific study has shown reduces cholesterol. This trend is also evident in reduced sales of sulia, a food that few of the most healthy citizens regularly eat.

Write a response in which you discuss what specific evidence is needed to evaluate the argument and explain how the evidence would weaken or strengthen the argument.




The citizens of Forsythe have adopted more healthful lifestyles. Their responses to a recent survey show that in their eating habits they conform more closely to government nutritional recommendations than they did ten years ago.

Claim:The citizens of Forsythe have adopted more healthful lifestyles for results from a survey.
Assumption: Responses can represent all citizens
Questions
-Who response to the survey and how they response it should be known .

Furthermore, there has been a fourfold increase in sales of food products containing kiran, a substance that a scientific study has shown reduces cholesterol.
Claim: Sales of kiran concluded products spurs which means people are looking for it.
Assumption: Sales can represent people’s intention on shopping.
Questions
-There are many reasons for people buying certain products , without further information such as a survey about why people buying it , we cannot draw such conclusion .
-Eating a certain food is not enough to be the standard for healthy lifestyle .There are other things for a healthy lifestyle such as working out and their whole diet .

This trend is also evident in reduced sales of sulia, a food that few of the most healthy citizens regularly eat.
Claim: Sales of sulia reduced means citizens of Forsythe have adopted more healthful lifestyles.
Assumption : Healthy people can be the standard to decide if a food is healthy or not.
Question:
-We need more information about sulia , since we don’t know if sulia is harmful to one’s health .
-Even if sulia is harmful to health , we still don’t know why these healthy people abandon it from their diet . Maybe it’s simply because of its taste.


TS: In this article , the author claim that citizens of Forsythe have adopted more healthful lifestyles, To support this recommendation the argument point out that a recent survey show that in their eating habits they conform more closely to government nutritional recommendations and increase in sales of food that contains kiran and sales of a food that few of the most healthy citizens regularly eat called sulia reduced. However the author’s reasoning is incomplete unless additional information are given. Without certain data we can’t evaluate author’s reasoning line and draw a conclusion.

Pt1:  To start with, we need to consider the fundamental question whether the survey the author provides is valid. Since we are not given any information about the scope of the survey or the constituent of respondents, it is possible that the survey is biased, or statistically invalid at all. For example, the author didn’t mention who took part in the survey there is a possibility that only healthy people response to it .If so , few healthy people’s attitude can’t represent every citizen in the Forsythe. Moreover, information about how the survey really is are quite limited . if the survey is nothing but asking people questions , the reliability of the results are doubted for we can’t guarantee everyone tell the truth. Also , author compare these data to the data ten years ago, but we don’t have evidence about if they are the same people. There is a possibility people who are invested now are no longer the same people who provide data ten years ago .


作者: envy213    时间: 2017-8-17 21:30:21

53) The following appeared in a health magazine.


The citizens of Forsythe have adopted more healthful lifestyles. Their responses to a recent survey show that in their eating habits they conform more closely to government nutritional recommendations than they did ten years ago. Furthermore, there has been a fourfold increase in sales of food products containing kiran, a substance that a scientific study has shown reduces cholesterol. This trend is also evident in reduced sales of sulia, a food that few of the most healthy citizens regularly eat.


Write a response in which you discuss what specific evidence is needed to evaluate the argument and explain how the evidence would weaken or strengthen the argument


Claim1: The citizens of Forsythe have adopted more healthful lifestyles.


data1: Their responses to a recent survey show that in their eating habits they conform more closely to government nutritional recommendations than they did ten years ago


Warrant1: nutritional eating habits mean healthful lifestyles


Claim2:The eating habits of citizens have been improved


data2:there has been a fourfold increase in sales of food products containing kiran, a substance that a scientific study has shown reduces cholesterol.


Warrant2:increasing sales of healthy food reflect that citizens adopt healthier eating habits.


claim3:citizens eat healthier than before


data3:This trend is also evident in reduced sales of sulia, a food that few of the most healthy citizens regularly eat


Warrant1: healthy citizens don’t eat sulia, the unhealthy food



Introduction
According to the article in a health magazine, the The citizens of Forsythe have adopted more healthful lifestyles.The author based his assumption on the survey that
the citizens of Forsythe conform more closely to government nutritional recommendations than they did ten years ago,which is reflected on the sales of food products--sales of heathy foods have risen while sales of unhealthy foods have declined.However, the assumption should be taken with a salt of pinch because a healthy lifestyle is not just about eating habit but also about various factors,so is the sales of food.

Middle part 1
First and foremost, it is assumed that The citizens of Forsythe have adopted more healthful lifestyles. Though the survey reflect that the citizens' eating habits conform more closely to government nutritional recommendations than they did ten years ago, a healthy lifestyle is a product of various factors. Maybe the citizens eat healthier than before while also exercise less ,which must be counterproductive to the development of healthy lifestyle.Hence, the citizens may not adopt more healthful lifestyles.
作者: xiulin2015    时间: 2017-8-17 21:31:23

151) Benton City residents have adopted healthier lifestyles. A recent survey of city residents shows that the eating habits of city residents conform more closely to government nutritional recommendations than they did ten years ago. During those ten years, local sales of food products containing kiran, a substance that a scientific study has shown reduces cholesterol, have increased fourfold, while sales of sulia, a food rarely eaten by the healthiest residents, have declined dramatically. Because of these positive changes in the eating habits of Benton City residents, we predict that the obesity rate in the city will soon be well below the national average.

Write a response in which you discuss what questions would need to be answered in order to decide whether the prediction and the argument on which it is based are reasonable. Be sure to explain how the answers to these questions would help to evaluate the prediction.


claim: It is predicted that the obesity rate in the obesity rate in the Benton city will soon be well below the national average because of the positive changes in the eating habits of its residents.
data: food products containing kiran, reducing cholesterol, have increased fourfold.  
data: sales of sully have decline dramatically.
warrant: the eating habits of the city residents is confirmed to be more closely to government nutritional recommendations than they did ten years ago.

Introduction Paragraph:
This argumentation predicts the obesity rate in Benton city will soon be lower than the national average due to the positive changes in the residents’ eating habits. However the questions to this prediction are whether the food products containing kiran and sulia have any relations to the obesity.

Middle Paragraph
The recent survey shows local sales of food products containing kiran have increased fourfold and it is regarded as a substance to reducing cholesterol. It is not quite clear whether that food is helpful to control the obesity and maybe the cholesterol is originally not that high in the bodies of the residents here. If there are no surveys to show the changes of cholesterol level 10 years ago and after, the increasing of kiran food couldn’t be taken as the positive change to decrease the obesity rate. The other possible scenario is the increased sales of Kiran food are not consumed by most of the residents in Benton city but only a few residents who actually have higher demand on this sort of food products than 10 years ago.Then it is very necessary to investigate what are the food eaten daily by most people living in Benton City and whether the cholesterol rate has dropped hugely in comparison of that in 10 years ago.

The survey also shows the declining of sales of sulia in the last decade and it is believed as a positive change to decrease obesity as this food is rarely eaten by the healthiest residents. As such, it is very conspicuous whether these residents are healthiest due to the deficiency of sulia. It could be for many reasons for them to keep healthy and become the healthiest but not happen to be only for one food. Furthermore there is no strong evidence to indicate sulia will increase obesity as there is no survey to demonstrate sulia appeared most frequently on the food menu of the patients with obesity. Therefore to clarify whether less sulia would contribute positively to the lower obesity rate needs more researches on the food menus of obesity residents and differences from the healthiest people in dinning and living habits.         
作者: 前意识的斑点    时间: 2017-8-17 21:46:05

144) The citizens of Forsythe have adopted more healthful lifestyles. Their responses to a recent survey show that in their eating habits they conform more closely to government nutritional recommendations than they did ten years ago. Furthermore, there has been a fourfold increase in sales of food products containing kiran, a substance that a scientific study has shown reduces cholesterol. This trend is also evident in reduced sales of sulia, a food that few of the healthiest citizens regularly eat.

Write a response in which you examine the stated and/or unstated assumptions of the argument. Be sure to explain how the argument depends on these assumptions and what the implications are for the argument if the assumptions prove unwarranted.


Claim: The citizens of Forsythe have adopted more healthful lifestyles.
Data: There is a recent survey shows that in their eating habits they conform more closely to government nutritional recommendations than they did ten years ago.
Warrant: This survey is good enough to show the reality of the citizens of Forsythe

Assumption: This survey is good enough to show the reality of the citizens of Forsythe

Claim: The citizens of Forsythe have adopted more healthful lifestyles.
Data: there has been a fourfold increase in sales of food products containing kiran, a substance that a scientific study has shown reduces cholesterol.
Warrant: There are increasing number of people start to buy healthy food products


Claim: The citizens of Forsythe have adopted more healthful lifestyles.
Data: This trend is also evident in reduced sales of sulia, a food that few of the healthiest citizens regularly eat.
Warrant: There are a decreasing number of people that are willing to buy sulia because they prefer healthier food.

Introduction:
In this argument, the author states that the individuals of Forsythe are used to more healthful lifestyles. In order to strengthen the argument, the author makes bold assumptions about a recent survey, the sales rate of healthy and unhealthy food products. However, this does not constitute a logical argument in favor of its conclusion and fails to provide convincing support making the argument sound.

Middle:
To begin with, it is assumed that the recent survey can mirror the reality of the individuals of Forsythe. The survey might not take a lot of factors into account, such as genders, ages as well as accuracy. For example, the female of Forythe were far more into healthy diet than the male. Thus, if the institution that performed the survey had more female samples than male samples, the result would not be accurate enough. What's more, the social background of the survey sample is unknown. Maybe all the rich people of Forsythe were into healthier diet than they did ten years ago because they had the budget. However, the middle class and below of Forsythe were still not capable of affording this kind of lifestyle. As a result, if the survey only samples from a certain group of people, it could not reflect the reality.

Even if the survey is strong enough to mirror the truth, the author fails to draw out the connection between kiran and healthy lifestyle. H/She assumes that the sales of food products containing Kiran went up because people were into healthy food. Admittedly, kiran is a healthy ingredient itself. However, it is possible that the food products containing Kiran are actually not healthy. Most of them contain a lot of fat and oil. Individuals bought them mainly because of the taste.

Similarly, the author assumes that the sales of sulia reduced because healthy lifestyles are trendy this day. However, the reason why the healthiest citizens did not eat it is unknown. Maybe the food itself contains a lot of proteins as well as vitamins, but it does not taste good. In fact, the cooking method for this ingredient might extremely complicated. Or, farmers put a lot of pesticides when sulia are growing. As a result, individuals stop purchasing it because they were scared of pesticides. Thus, it can not showcase that people stopped purchasing it because of the trend.

In conclusion, the writer states that more individuals of Forsythe were into healthful lifestyles. However, the survey and the sale rate of certain food products might not be strong enough support this argument.
作者: listeningwutt    时间: 2017-8-17 21:48:42

本帖最后由 listeningwutt 于 2017-8-17 22:24 编辑

144) The citizens of Forsythe have adopted more healthful lifestyles. Their responses to a recent survey show that in their eating habits they conform more closely to government nutritional recommendations than they did ten years ago. Furthermore, there has been a fourfold increase in sales of food products containing kiran, a substance that a scientific study has shown reduces cholesterol. This trend is also evident in reduced sales of sulia, a food that few of the healthiest citizens regularly eat.
Write a response in which you examine the stated and/or unstated assumptions of the argument. Be sure to explain how the argument depends on these assumptions and what the implications are for the argument if the assumptions prove unwarranted.

Claim: Following the government nutritional recommendation will lead to a healthful lifestyles.
Data: Recent survey suggests that the eating habits of citizens are more conformed to the government nutritional recommendation than 10 years before.
Warrant: government nutritional recommendation is good.

Claim: Eating more iran will lead to health.
Data: The increasing in the sales of food containing kiran
Warrant: kiran can reduce cholesterol.

Claim: Eating less sulia will lead to health.
Data: reduce sales of sulia
Warrant: the healthiest citizens regularly don’t eat sulia

According to the reading material, the lifestyles of the citizens of Forsythe have become much healthier, because of the surveys conducted on the eating hats and the food sales. Several assumptions have been made about the government nutritional recommendation, kiran, and the sulia, in develop the argument. However, these assumptions need to be scutinized critically to assess the conclusion.

To begin with, the author assumes that the government nutritional recommendations is good. Thus when citizens are more attached to these recommendations, they will have a good heath. However, recommendations may be made by some unprofessional persons, which means they cannot guarantee for health or even leads to being harmful for bodies. If recommendations were not healthy or effective, the citizens’ wouldn’t have become healthier.

点评37楼:
我觉得你的三个切入点都是好的
但是每个切入点中的展开有一些问题,不是很清楚。
比如说在调查人背景不同这一点,你提到了富人和穷人的人生健康可能变化不一样,那么之后你应该提,这个调查把目标人群都锁定在上流阶级了,所以调查上虽然是健康了,但不代表整个社会的水平。
再者,你的这一段背景这里,只提到上中流人群金钱背景不一样,之后也要提到这导致他们是否有能力去按着镇府建议的食谱饮食。要把材料里的 recommendations 加到你的推导里
作者: 前意识的斑点    时间: 2017-8-17 21:57:57

listeningwutt 发表于 2017-8-17 21:48
144) The citizens of Forsythe have adopted more healthful lifestyles. Their responses to a recent su ...

看了一下哈~,感觉第一个warrant有点不对,不是应该看assume了啥咩,
第二三个不是应该看本质咩

感觉看了文章有一点语法错误,比如in develop the argument,然后感觉下边的第一段的例子不够有说服力~应该说这个研究有问题,可能会更好

作者: alicechun95    时间: 2017-8-17 22:22:23

Arg 1 Palean Basket unique

Woven baskets characterized by a particular distinctive pattern have previously been found only in the immediate vicinity of the prehistoric village of Palea and therefore were believed to have been made only by the Palean people. Recently, however, archaeologists discovered such a 'Palean' basket in Lithos, an ancient village across the Brim River from Palea. The Brim River is very deep and broad, and so the ancient Paleans could have crossed it only by boat, and no Palean boats have been found. Thus it follows that the so-called Palean baskets were not uniquely Palean.

Write a response in which you discuss what specific evidence is needed to evaluate the argument and explain how the evidence would weaken or strengthen the argument.

Claim: ancient Paleans could have crossed the Brim River only by boat
Data: Lithos is an ancient village across the Brim River from Palea, and the river is very deep and broad.
Warrant: the river is deep and broad now, so it was the same in the past.

Evidence: the accurate state of this river at the same time as woven baskets, records about the development of the Brim River

claim: Paleans did not invent boat at that time (implied)
data: no Palean boats have been found
warrant: if boats are not found now, they don’t exist in the past.

evidence: records or documents about boats at that time, if there are any limitations on the excavation of relics in the local area; or the evidence that the boats were destroyed before.

Claim: The basket found in Lithos is believed to be produced by the local people.(implied)
data: archaeologists discovered such a 'Palean' basket in Lithos
Warrant:  the baskets were invented by the people living in the place where they were found.

Evidence: other documents to prove the baskets are invented by Lithos, maybe it was just taken to Lithos by someone


开头段:
According to the statement, Palean baskets are not uniquely Palean due to the fact that archaeologists have found a similar one in Lithos. Given the fact that there’s a deep and broad river between Palea and Lithos and no remainings of Palean boats have been found by scientists, it is believed that maybe people in Lithos also invented the same baskets. However, evidences on the river, boat and basket inventor need to be assessed in order to make the statement sound.




作者: 露葵.    时间: 2017-8-17 22:33:47

本帖最后由 露葵. 于 2017-8-19 15:26 编辑

Arg 1 Palean Basket unique

Woven baskets characterized by a particular distinctive pattern have previously been found only in the immediate vicinity of the prehistoric village of Palea and therefore were believed to have been made only by the Palean people. Recently, however, archaeologists discovered such a 'Palean' basket in Lithos, an ancient village across the Brim River from Palea. The Brim River is very deep and broad, and so the ancient Paleans could have crossed it only by boat, and no Palean boats have been found. Thus it follows that the so-called Palean baskets were not uniquely Palean.
思路:河在那时存在吗=>河存在的话档得住人吗=>河挡得了人,那没找到船一定代表没船吗。有没有桥


claim: The Palean baskets were origined both Palean and Lithos.
data: The Palean baskets were discovered in Lithos, where the Palean people could not reach by boat.
assumption1: The only way to cross the river for Palean people is taking a boat.
The river really exist at Palean period.
The river has been deep and broad from ancient time.
There was no periods when the river was frozen.
No brigdes built.
No other roads ever existed.
evidence1: More information about the  paleoclimate and paleotopography.


claim:There were no boats in the history of Platean.
assumption: If no boats were found in the remains, there were no boats at all.
evidence:  

文章: 前面那个cycle有点乱哈,看文章应该思路更清楚
开头:
While it may be true that Palean baskets were not uniquely, the author fails to analyze other possible scenarios to fully evaluate this case. According to the argument, Palean baskets were found in Lithos, across a river from Palean. In addition, no boats were found in Palean, and thus, the authors concludes that the Palean baskets were not uniquely Palean. Nevertheless, it's necessary to offer more information about the existence of the river at ancient time, the climate and tectonic movement of Palean and the technologies of Palean people.

To begin with, it's assumed that the Palean people couldn't go across the Brim river on the account of its depth and width. However, there is no evidence that the river had already existed at that time. It's possible that a tectonic motion like an earthquake produced a river between Palean and Lithos after Palean period. In this case, the Palean people were able to bring their baskets to Lithos by just walking. So evidence of the existence of Brim river at Palean period should be offered.

Secondly, even if the river did exist at that time, it doesn't necessarily mean the river can prevent people from going across it with no boats. Consider the possibility that Brim river is not as deep as it is now, or that the climate is so much colder than today that the water was frozen every winter. balabala....
(need evidence about the climate and terrain of Palean )

Granted that people cannot go to the other side without a boat because of the river, still, another important evidence we need is about the technologies of Palean people. Given no Palean boats have been found, the author reached a conclusion that there were no boats built in Palean. However,..balbala
(need evidence that they couldn’t build bridge and that the main material they could have used will remain until today )

结尾:In a nutshell, considering all the possible scenarios above, the argument is only  one of the posibilities, and more specific evidences are required to make a conclusion.

疑问:各位~小白求解答!!

写作指引里有一句:explain how the evidence would weaken or strengthen the argument,我这里写了需要的evidence,但是感觉好像没怎么写它将对argument造成什么影响,请问要怎么改来贴合这个指引?还是说有前文指出作者思维漏洞,another scenario 就可以啦?
另外,我看到很多人说这种一段一段让步even/even though/granted that 写法XDF很推崇,会不会造成大家都这样写,非常模板化,导致分数很低呢。但是我确实觉得这样让步很有道理啊。。。   求解答~谢谢!

批改:14楼
三个攻击点是准的

中见段的第二段,中写售卖总量变多但是人均变少,可是argument中写的是增加四倍哦,人口激增四倍还是不太可能吧。我觉得不如写含有kiran产品的其他添加剂不明确,需要知道这些产品是否高糖,高油脂

结尾段,food containing in daily life 似乎与第二第三个论点不对应,还有instead of assumption or the laboratory. 应该是lab results吧,还有lab 的结果应该是作者assumption 的一部分才对。
作者: taylor1995    时间: 2017-8-17 22:40:27

151) Benton City residents have adopted healthier lifestyles. A recent survey of city residents shows that the eating habits of city residents conform more closely to government nutritional recommendations than they did ten years ago. During those ten years, local sales of food products containing kiran, a substance that a scientific study has shown reduces cholesterol, have increased fourfold, while sales of sulia, a food rarely eaten by the healthiest residents, have declined dramatically. Because of these positive changes in the eating habits of Benton City residents, we predict that the obesity rate in the city will soon be well below the national average.

Write a response in which you discuss what questions would need to be answered in order to decide whether the prediction and the argument on which it is based are reasonable. Be sure to explain how the answers to these questions would help to evaluate the prediction.

claim:  the obesity rate in the city will soon be well below the national average.
data:1.the increasement of local sales of food products containing kiran
        2.the sales of sulia, a food rarely eaten by the healthiest residents, have reduced
Warrant: citizens eat more healthy food and less unheathy food.

Intro
According to a recent survey, the Benton citizens’ eating habits are more close to the government healthy suggestion than before. Given the fact that some positive changes of eating habits were kept, the writer argues that the obesity rate in Benton will decrease and be below the native average rate. Many assumptions are put forward in this argument, but it should be proved to strengthen this argument.

Middle
First, the conclusion come from a recent survey, but there is no data to show how many people join this survey which loses the reliability. Second, the writer believed that the increased sales of the food including kiran which decline the cholesterol indicated more citizens change to healthy life style. However, such a fact does not necessarily imply healthiness. It may be not only good for reducing the cholesterol but the cheaper price. Then, another assumption is the reduction of sulia, because of the reduction in heathiest citizens. There is no evidence to prove sulia is unfavorable to people and what kind of people buy it. In conclusion, it not adequate to obtain this argument.
作者: MaggieLLLLLL    时间: 2017-8-17 22:41:18

151) Benton City residents have adopted healthier lifestyles. A recent survey of city residents shows that the eating habits of city residents conform more closely to government nutritional recommendations than they did ten years ago. During those ten years, local sales of food products containing kiran, a substance that a scientific study has shown reduces cholesterol, have increased fourfold, while sales of sulia, a food rarely eaten by the healthiest residents, have declined dramatically. Because of these positive changes in the eating habits of Benton City residents, we predict that the obesity rate in the city will soon be well below the national average.

Write a response in which you discuss what questions would need to be answered in order to decide whether the prediction and the argument on which it is based are reasonable. Be sure to explain how the answers to these questions would help to evaluate the prediction.
       
Claim: Benton City residents have adopted healthier lifestyles.
Data: A recent survey of city residents shows that the eating habits of city residents conform more closely to government nutritional recommendations than they did ten years ago.
Warrant: The survey is reliable

Claim: Benton City residents have adopted healthier lifestyles.
Data: During those ten years, local sales of food products containing kiran, a substance that a scientific study has shown reduces cholesterol, have increased fourfold
Warrant: The rising sales of products containing kiran means that people are living more healthily

Claim: Benton City residents have adopted healthier lifestyles.
Data: sales of sulia, a food rarely eaten by the healthiest residents, have declined dramatically.
Warrant: the declining sales of sulia is the result of the healthier lifestyle of Benton’s citizens

Claim: The obesity rate of the city will soon below the national average
Warrant: Benton City residents have adopted healthier lifestyles.

The author of this argument reached a conclusion that the obesity rate of Benton City will soon be below the national average as, proposed by the author that, Benton’s citizens have developed a healthier lifestyle. To support this argument, the author addressed 3 specific evidence that a recent survey shows people’s eating habits skewed more towards national recommendation, and that the sales of the products containing kiran has increased while the sales of sulia has decreased. In order to examine the validity of the argument, however, we still need to ask more questions, which are whether the survey is reliable and representative, whether the increasing sales of food containing kiran and dropping sales of sulia are reliable indicators of healthier lifestyle.

Firstly, we need to ask more questions about the survey. Are the questions well-designed to avoid any misunderstanding among its respondents? Are the questionnaire distributed through proper channels that would make everybody has the equal chance to participate in? As the development and advancement of the computers and mobile phones has increased dramatically, more and more questionnaires started to be distributed through online channel, though really convenient, would have the problems when the voice of people who don’t have the access to the internet may be ignored. Also, are the comparison of eating habits between what people currently have and ten years ago reasonable? Do they do the study longitudinally or simply ask people to recall what they eat ten years ago. If it is simply a recall, the reliability of the survey results would be undermined as it relies on people’subjective memories which would be mistaken or twist.

作者: xiulin2015    时间: 2017-8-17 22:57:24

TaylorW 发表于 2017-8-17 21:29
53) The following appeared in a health magazine.

The citizens of Forsythe have adopted more healt ...

your middle paragraph is impressive as it is a holistic review on the research structure and how the research is conducted. However the middle paragraph is supposed to review and elaborate on one particular aspect in the argumentation. Hence I am not sure whether it is more appropriate to put your general review at last or put it as part of the introduction paragraph. Moreover, the test requires what specific evidence is needed to evaluate the argument and it might be missing from your middle paragraphs. For your reference only. Appreciate your point of view on samples, the change of samples through the decade.     
作者: Leona2017    时间: 2017-8-17 22:58:02

Arg 1 Palean Basket unique

Woven baskets characterized by a particular distinctive pattern have previously been found only in the immediate vicinity of the prehistoric village of Palea and therefore were believed to have been made only by the Palean people. Recently, however, archaeologists discovered such a 'Palean' basket in Lithos, an ancient village across the Brim River from Palea. The Brim River is very deep and broad, and so the ancient Paleans could have crossed it only by boat, and no Palean boats have been found. Thus it follows that the so-called Palean baskets were not uniquely Palean.

Write a response in which you discuss what specific evidence is needed to evaluate the argument and explain how the evidence would weaken or strengthen the argument.

Claim:Woven baskets characterized by a particular distinctive pattern were believed to have been made only by the Palean people
Data:They are found only in the immediate vicinity of the prehistoric village of Palea
Warrant: If something is found only in a certain area, the thing is unique in that place.

Claim: Paleans could have crossed it only by boat
Data:The Brim River is very deep and broad
Warrant:Boating is the only way for people at the time to cross the river
Question: What if there’s an unfound road that connects B&L under the river?

Claim:The so-called Palean baskets were not uniquely Palean
Data: No Palean boats have been found
Warrant:If the boats had been built, they must be found
Question: What if the boats are destroyed or sink?

According to this argument, the Palean baskets are not unique in that area since they were also found in another village nearby. However, to strengthen the soundness of this argument, more evidence is required. We need to know more about the geographical information of the area, the record of the traffic between the two villages and the materials of the baskets.

We need to know more about the geographical information of the area. In other words, it is possible that there had been a path that connected Palea and Lithos but it was covered by the deep water so has not been discovered yet. If the path existed, then people from Palea could simply go to Lithos on foot instead of by boat and therefore, the baskets could be brought to another place without any boats.
作者: bio-L    时间: 2017-8-17 23:09:53

Arg 1 Palean Basket unique

Woven baskets characterized by a particular distinctive pattern have previously been found only in the immediate vicinity of the prehistoric village of Palea and therefore were believed to have been made only by the Palean people. Recently, however, archaeologists discovered such a 'Palean' basket in Lithos, an ancient village across the Brim River from Palea. The Brim River is very deep and broad, and so the ancient Paleans could have crossed it only by boat, and no Palean boats have been found. Thus it follows that the so-called Palean baskets were not uniquely Palean.

Write a response in which you discuss what specific evidence is needed to evaluate the argument and explain how the evidence would weaken or strengthen the argument.

claim   the ancient Paleans could have crossed it only by boat
data  The Brim River is very deep and broad
warrant the river's condition means that.
evidence  the condition of ancient Brim river

claim  the palean basket was made by lithos people.
data no Palean boats have been found
warrant  palean basket can only be transferred by boat, and no palean boat at that time,so
'Palean' basket in Lithos cannot be transferred from palean to lithos
evidence  if any other way still existed to transfer the palean basket, like an accident.
              if lithos had boat to cross the river.

claim (ancient plean had no boat  )
data no boat was found
warrant   no boat was found means plean couldn't have boat in the past.
evidence   if the boat could be preserved till now.

intro
The author argues that Palean baskets were not uniquely made in Palean as another palean basket was found in Lithos, and the evidence including the local condition of palean and no boat was found manifest that it cannot be made by palean. To evaluate the argument, we need more evidence which are mainly about the condition of ancient Brim river, the possibility of the boat preserves till now and other way to cross the river.

para 1
For one thing, the author doesn't indicate that the ancient Brim river had same aspect and condition with nowadays'. the author just shows the resent condition of Brim river, i.e., it's too broad and deep not to be crossed except by boat. However, the chance is that this river would be crowed and shadow and easily passed in the past. And maybe the river was even not exist at that time. In this situation, ancient palean people needn't to own a boat to arrive at lithos. So, we could get the conclusion that the palean people couldn't go to lithos only because no boat have been found in palean. If some evidence approve the assumption above,then the palean blacket have the chance to be sent to lithos, and the author's argument will be undermined strongly.

作者: fh_seu    时间: 2017-8-17 23:16:03

144)
The citizens of Forsythe have adopted more healthful lifestyles. Their responses to a recent survey show that in their eating habits they conform more closely to government nutritional recommendations than they did ten years ago. Furthermore, there has been a fourfold increase in sales of food products containing kiran, a substance that a scientific study has shown reduces cholesterol. This trend is also evident in reduced sales of sulia, a food that few of the healthiest citizens regularly eat.

Write a response in which you examine the stated and/or unstated assumptions of the argument. Be sure to explain how the argument depends on these assumptions and what the implications are for the argument if the assumptions prove unwarranted.

Claim: The citizens of Forsythe have adopted more healthful lifestyles.
Data: Their responses to a recent survey show that in their eating habits they conform more closely to government nutritional recommendations than they did ten years ago.
Warrant: (having an eating habit which is close to the government recommendation means a healthier lifestyle) implied
Assumptions:
1)        The survey is persuasive and objective
2)        The survey did ten years ago is credible
3)        The eating habit which is close to the government recommendation is healthier

Claim: The citizens of Forsythe have adopted more healthful lifestyles.
Data: there has been a fourfold increase in sales of food products containing kiran, a substance that a scientific study has shown reduces cholesterol
Warrant: (the increase in the sales of food products containing kiran in Forsythe indicates the healthful lifestyles of citizens) implied
Assumptions:
1)        It is people’s concern about healthful lifestyles that leads the increasing sales of healthful food products, such as the one containing kiran
2)        The increase in the sales of food products containing kiran is significant enough in amount and benefited many of the citizens
3)        The reduction of cholesterol means healthful

Claim: The citizens of Forsythe have adopted more healthful lifestyles.
Data: the reduced sales of sulia, a food that few of the healthiest citizens regularly eat.
Warrant: The decline in sales of suila has proven the trend of advocating healthy lifestyles in Forsythe
Assumption:
1)        The foods that few of the healthiest citizens regularly eat are unhealthful
2)        The decline in sales of suila shows people's attention to health

Introductory paragraph
The author argues that the citizens of Forsythe have adopted more healthful lifestyles. A number of assumptions have been made about the recent survey concerning eating habits of citizens, and the sales records of two types of food. We need to examine these assumptions carefully to decide if the argument is reasonable.

Middle paragraph
To begin with, it is assumed that the surveys about citizens’ eating habits could draw the conclusion that the city residents adopted more healthful lifestyles than they did ten years ago. It’s not clear, however, the scope and validity of that survey. For one thing, could these sampled respondents truly represent the Forsythe citizens? If the respondents were mainly from a particular socioeconomic class or the amount of respondents was not big enough, for example, the survey results could not be generalized to the entire population in Forsythe. For another thing, is the eating habit which is close to the government recommendation healthier? As we all known, the standard for a healthful lifestyle is not unique. Even if this survey could prove that people's eating habits are closer to the government nutritional recommendations, the author cannot say that people's lifestyles are healthier without scientific research. Unless the survey is fully representative, valid, and reliable, it cannot be used to effectively back the authors’ argument.
作者: 遠子w    时间: 2017-8-17 23:24:05

Arg 1 Palean Basket unique

Woven baskets characterized by a particular distinctive pattern have previously been found only in the immediate vicinity of the prehistoric village of Palea and therefore were believed to have been made only by the Palean people. Recently, however, archaeologists discovered such a 'Palean' basket in Lithos, an ancient village across the Brim River from Palea. The Brim River is very deep and broad, and so the ancient Paleans could have crossed it only by boat, and no Palean boats have been found. Thus it follows that the so-called Palean baskets were not uniquely Palean.

Write a response in which you discuss what specific evidence is needed to evaluate the argument and explain how the evidence would weaken or strengthen the argument.

Claim:         The so-called Palean baskets are not uniquely produced in Palean but also produced in other places
Data: Archaeologists discovered such a 'Palean' basket in Lithos
Warrant: The 'Palean' basket found in Lithos was made by Lithos people

Claim:         The so-called Palean baskets are not uniquely produced in Palean but also produced in other places
Data: The Brim River is very deep and broad, and so the ancient Paleans could have crossed it only by boat
Warrant: Therefore the 'Palean' basket found in Lithos could not be brought to Lithos by Palean people without boat. Instead, it was more likely produced in Lithos

Claim:         The so-called Palean baskets are not uniquely produced in Palean but also produced in other places
Data: No Palean boats have been found
Warrant: Paleon people do not use boats for travel and trade since Palean boats have not been found.

Based on the recent archaeological finding, the author of the argument argues that the woven baskets, which were only found in Palean in the past, are not uniquely produced in Palean but probably also in other places such as Lithos. However, in order to evaluate the argument, we need more specific evidences such as the archaeological information about the basket, the depth of the river in the ancient time, the existence of bridge, and the existence of Lithos boats.

Firstly, the author need more archaeological information about the baskets to determine whether the woven baskets found in Lithos are originated from Lithos or are recently being brought to Lithos. Baskets recently found in Lithos need to be examined. For example, if there were signatures or specific signs by Palean people on the baskets, we may conclude that the baskets might be made in Palean, and the basket might be transported to Lithos in the past by means other than boats or transported recently. Thus, the argument can be refuted in this case. If there were signatures by Lithos people, the baskets in Lithos might be made in Lithos. Therefore, the author’s argument can be supported by these evidences in this case. More information regarding the issue is needed.


作者: LynnHan    时间: 2017-8-17 23:27:15

53)The following appeared in a health magazine.

The citizens of Forsythe have adopted more healthful lifestyles. Their responses to a recent survey show that in their eating habits they conform more closely to government nutritional recommendations than they did ten years ago. Furthermore, ther ce that a scientific study has shown reduces cholesterol. This trend is also evident in reduced sales of sulia, a food that few of the most healthy citizens regularly eat.

Write a response in which you discuss what specific evidence is needed to evaluate the argument and explain how the evidence would weaken or strengthen the argument.

思路:三点都在assume市民的饮食习惯变好了,从而推出more healthful lifestyle,先从这点开头。然后分述三个data。画了个图。。好像传不上


开头段:As is presented in the health magazine,the writer believes that Forsythe’s residents have become more healthier since they seem to adopt better eating habits.While this assumption might be conceivable to some extent,the reasoning of deduction process is unpersuasive due to several unsubstantiated evidence which,is proven not exactly,will seriously challenge the author’s conclusion.

中间段1:First of all,the writer’s conclusion relies heavily on the assumption that there are no alternative factors related to a fitness living module.In other words,people are bound to live a healthier life if they just concern about what they eat.However,this assumption is potentially problematic because we are not informed whether the citizens of Forsythe have perform well in other scenario,such as exercising frequency,sanitation habits,etc. If it turns out that their nutrition consumption are progressive but show negligence to basic work-rest habits,then the author’s assumption for trends of healthier lifestyle is invalid.

作者: Fitness-Gu    时间: 2017-8-17 23:27:20


According to the author, the obesity rate of Benton city is projected to be below the domestic average because of the improvement of eating habits of citizens. Citing to comparisons of citizens’eating habits to government recommendations and data about sales of kiran and sulia, the writer tries to prove it. A number of questions must be answered in order to evaluate the argument, such as whether the government nutritional recommendations were healthier than the eating habits citizens had ten years ago, whether the survey is scrutinized,whether healthy food becomes more welcome recently and what different factors can lead to the obesity rate above average. Without precise answers,we cannot conclude the obesity rate will drop in the near future.

To begin with,we should know whether Benton city residents adopt healthier eating habits than before. Only that they are eating healthier, we can believe that the obesity rate may decrease. Though the survey claim that their eating habits conform more closely to government nutritional recommendations, we do not get any information about details of these recommendations so that we can’t draw a conclusion that people’s healthier lifestyles result from their adopting to government recommdations. In addition, we need to know whether the eating habits ten years ago were less healthier than current nutritional recommendations. If not, the arguement that when people adopt to government nutritional recommdations,they will become more healthier may not be accurate and we still cannot come to a conclusion of the decrease of obesity rate in the future. Given the recommendations are standard of healthy diet and eating habits ten years ago are less healthy than it, the author needs to answer questions about the reliability about the survey, such as how many residents have been investigated, can those participants reflect the eating habits on average and how to decide whether their eating habits are close to recommendations. Without the answers to these questions, the survey and any conclusions from that may not be reliable.

作者: 深井冰233    时间: 2017-8-17 23:31:45

53) The following appeared in a health magazine.

The citizens of Forsythe have adopted more healthful lifestyles. Their responses to a recent survey show that in their eating habits they conform more closely to government nutritional recommendations than they did ten years ago. Furthermore, there has been a fourfold increase in sales of food products containing kiran, a substance that a scientific study has shown reduces cholesterol. This trend is also evident in reduced sales of sulia, a food that few of the most healthy citizens regularly eat.

Write a response in which you discuss what specific evidence is needed to evaluate the argument and explain how the evidence would weaken or strengthen the argument.

claim: people's eating habits are more closely to government nutritional recommendations
data: Their responses to ...
warrant: survey results are confident
evidence needed: sample size
question: Are people absolutely honest during interviews?

claim: people are buying more healthy food products
data:Furthermore, ...
warrant: people are buying more food products containing kiran which reduces cholesterol
assumption: People buy them because of advertisement.
question: Having one kind of healthy food in a diet is not equal to having a healthy diet.

claim: people are buying less unhealthy food
data: This trend is also ...
warrant: people are buying less sulia which is unhealthy
question: Avoiding one kind of unhealthy food in a diet is not equal to having a healthy diet (avoiding all kinds of unhealthy food).

Introduction:
According to the health magazine, the citizens of Forsythe should have adopted more healthful lifestyles. Citing the results of a recent survey and figures of food sales, this magazine argues that people of Forsythe are eating more healthily. However, the listed evidence need to be scrutinized critically to assess the strength of the argument.

First body paragraph:
To begin with, it is assumed that the results of the survey reflect that the citizens' eating habits have become more closely to the government's nutritional recommendations than that a few years ago. Nevertheless, this finding itself does not prove that they are keeping more healthy diets because the government's recommendations have been changing. So the people's eating habits are not compared with the same recommendations. In addition, people may pretend to live more healthy lives than they actually do during interviews. What's more, the sample size of the survey is not stated in the argument, which is key to the results' confidence level.
作者: 安歌    时间: 2017-8-17 23:35:44

Arg 53healthful lifestyles manifested in food sales

53) The following appeared in a health magazine.

The citizens of Forsythe have adopted more healthful lifestyles. Their responses to a recent survey show that in their eating habits they conform more closely to government nutritional recommendations than they did ten years ago. Furthermore, there has been a fourfold increase in sales of food products containing kiran, a substance that a scientific study has shown reduces cholesterol. This trend is also evident in reduced sales of sulia, a food that few of the most healthy citizens regularly eat.

Write a response in which you discuss what specific evidence is needed to evaluate the argument and explain how the evidence would weaken or strengthen the argument.

According to the reading, conclusion that citizens of Forsythe have adopted healthier lifestyles is drawn from a number of assumptions, which are based on a survey and the sales of kiran and sulia. However, a closer scrutiny of the assumptions would cast doubt on the soundness of the conclusion. Discussions are as follows.
In the first place, the conclusion is based on results of a recent survey, in comparison with results a decade ago. However, question remains that whether recipients gave the accurate response. As it is common in researches, interviewees might give false responses due to reasons such as social pressure, etc…..
In the second place, the reading contends that a fourfold increase in sales of food products containing kiran, a substance that reduces cholesterol, indicates that citizens are living a healthier lifestyle. However, possibility remains that products that contain healthy substances could also be unhealthy. …
Last but not least, though sales of sulia, a food that few healthy citizens regularly eat reduces, it is likely that the food could be healthy itself, only not meets the taste of healthy people.
In conclusion, it is difficult to confirm that Forsythe have adopted healthier lifestyles. The assumptions discussed all have its own weakness in logic. Therefore, it would require further evidence to draw a safe conclusion on the transformation of lifestyles among Forsythe citizens.

作者: 天地1沙鸥    时间: 2017-8-17 23:47:58

144) The citizens of Forsythe have adopted more healthful lifestyles. Their responses to a recent survey show that in their eating habits they conform more closely to government nutritional recommendations than they did ten years ago. Furthermore, there has been a fourfold increase in sales of food products containing kiran, a substance that a scientific study has shown reduces cholesterol. This trend is also evident in reduced sales of sulia, a food that few of the healthiest citizens regularly eat.

Write a response in which you examine the stated and/or unstated assumptions of the argument. Be sure to explain how the argument depends on these assumptions and what the implications are for the argument if the assumptions prove unwarranted.

Claim: healthy life than before
Data: Recent survey show that in their eating habits they conform more closely to government nutritional recommendations than they did ten years ago.
Warrants: 1) the eating habits are assessed under the assumption that old/new nutritional recommendations correspond to the people eating change
Assumption: 1) all the two recommendations reflects the eating habits corresponding to the time

Claim: healthy life than before
Data: a fourfold increase in sales of food products containing kiran which shows can reduce cholesterol by scientific study
Warrants: 1)The increase result from the desire of people to reduce the cholesterol  
Assumption: 1) people buy these food without no other purpose

Claim: healthy life than before
Data: reduced sales of sulia, a food that few of the healthiest citizens regularly eat.
Warrants: people do not eat the food of same kind  
Assumption: there is no other sources to get the food(self-made, alternative food)

开头段
According to the passage, the Forsythe citizens have lived more healthful lifestyles than before. Citing the recent survey and scientific study results, it argues that the change is mainly reflected on the eating habits. A number of assumptions have been made in the argument about the nutritional recommendations, food products containing kiran and sales of sulia. These assumptions need to be scrutinized critically to assess the strength of the argument.
中间段第二点
In addition, we also need to check the assumption that the sales increase of food containing sulia represents the lifestyles are more healthy. Maybe containing the sulia just make the food more delicious which results more people tend to buy them. Or perhaps the reason to buy these foods is just to follow the majority which is more attractive to people than reducing the cholesterol. If other factors contributes to the food sales increase, it may not matter the phenomenon shows the lifestyle is more healthy than before.

疑问:
1.warrants 跟 assumption 的区别感觉不是特别大,有的时候可以相近或相同(可能需要回顾视频)
2.感觉本题只有一个claim,那么每一段要不要直接采用相同的claim,或者即使相同claim是否需要换句式呢?
作者: SylviaMengyaya    时间: 2017-8-17 23:49:22

本帖最后由 SylviaMengyaya 于 2017-8-18 00:07 编辑

1) Woven baskets characterized by a particular distinctive pattern have previously been found only in the immediate vicinity of the prehistoric village of Palea and therefore were believed to have been made only by the Palean people. Recently, however, archaeologists discovered such a 'Palean' basket in Lithos, an ancient village across the Brim River from Palea. The Brim River is very deep and broad, and so the ancient Paleans could have crossed it only by boat, and no Palean boats have been found. Thus it follows that the so-called Palean baskets were not uniquely Palean.

Write a response in which you discuss what specific evidence is needed to evaluate the argument and explain how the evidence would weaken or strengthen the argument.

Claim: The ancient Paleans could have crossed it only by boat
Data: The Brim River is very deep and broad
Warrant: the Brim River has always been very deep and broad since that time; only by boat can people cross the Brim River.
Evidence: whether the Brim river is always deep and broad at that time; whether there was a bridge at that time.

Claim: the so-called Palean baskets were not uniquely Palean.
Data: The Brim River is very deep and broad and no Palean boats have been found
Warrant: no Palean boats have been found means that no boats have ever existed in this place.
Evidence: whether boats have ever existed in this area.

Intro:According to the argument, the Palean baskets ,once ware believed to have been made only by the Palean people, may have been made from areas other than Palae as they have also been found in an area close to Palae. We need more evidence about the geographic and climate information of the area, the information of bridge and boats building and transportation information at that time to evaluate the argument.

Middle:To begin with, we need information of climate and geography in that area at that time. Although it is assumed that the ancient Paleans could have crossed the Brim River only by boat as the river is very deep and broad, there could have other scenarios. For instance, maybe the river was nonexistent at that time. If the Brim River was nonexistent at that time, the Palaens could arrive other areas easily on their foot. On the other hand, if it was very hot and dry in the summer which would make the river become shallow at that time, people could walk across the river easily and trade the baskets.



作者: A.E.extrovert    时间: 2017-8-17 23:50:42

144)The citizens of Forsythe have adopted more healthful lifestyles. Their responses to a recent survey show that in their eating habits they conform more closely to government nutritional recommendations than they did ten years ago. Furthermore, there has been a fourfold increase in sales of food products containing kiran, a substance that a scientific study has shown reduces cholesterol. This trend is also evident in reduced sales of sulia, a food that few of the healthiest citizens regularly eat.

Write a response in which you examine the stated and/or unstated assumptions of the argument. Be sure to explain how the argument depends on these assumptions and what the implications are for the argument if the assumptions prove unwarranted.

The Central sentence: the citizens have adopted more healthful lifestyles.
Claim: people are being healthier than ten years ago.
Data: Their responses to a recent survey show that in their eating habits they conform more closely to government nutritional recommendations than they did ten years ago.
Warrant:the government nutritional recommendations is good for health, more closely to them means healthier lifestyles.

Claim: people are less likely to getting cholesterol because the food contain more kiran.
Data: Furthermore, there has been a fourfold increase in sales of food products containing kiran, a substance that a scientific study has shown reduces cholesterol.
Warrant: kiran can reduce cholesterol, and people eat lot of it can be healthier

Claim: people are healthier because they eat less sulia.
Data: This trend is also evident in reduced sales of sulia, a food that few of the healthiest citizens regularly eat.
Warrant:the healthiest citizens do not eat sulia regularly, and less people eat it means people have adopted a more healthier lifestyles.


Introductory:
The author argued that the citizens have adopted more healthful lifestyles. A number of assumptions have been made in the argument about the survey results,contained substance in the sales food and the kinds of food people eat. These assumptions needed to be scrutinized critically to assess the strength of the argument.

To begin with, it is assume that people are being healthier than ten years ago, as their eating habits are more conformed to government recommendations nutritional one than they did ten years ago.But the precondition of the claim is that diets the government recommended nowadays are healthier than ten years ago as the government may recommend diets no as healthy as before for the politic or commercial reasons which tender the public not healthier than before.If the diets the government suggested is not healthier, that a more healthful lifestyle may not be adopted by the citizens.
作者: keaixiaotang    时间: 2017-8-17 23:55:46

Arg 1 )Palean Basket uniqueWoven baskets characterized by a particular distinctive pattern have previously been found only in the immediate vicinity of the prehistoric village of Palea and therefore were believed to have been made only by the Palean people. Recently, however, archaeologists discovered such a 'Palean' basket in Lithos, an ancient village across the Brim River from Palea. The Brim River is very deep and broad, and so the ancient Paleans could have crossed it only by boat, and no Palean boats have been found. Thus it follows that the so-called Palean baskets were not uniquely Palean.

Write a response in which you discuss what specific evidence is needed to evaluate the argument and explain how the evidence would weaken or strengthen the argument.

Claim1 :the ancient Paleans could have crossed The Brim River only by boat
Data1:The Brim River is very deep and broad, and so the ancient Paleans could have crossed it only by boat,
Warrant 1:just because the river is so-called deep and broad, there is only one method which is boating to cross the river.

Claim2: the so-called Palean baskets were not uniquely Palean
Data2:Thus it follows that the so-called Palean baskets were not uniquely Palean.
Warrant2: just because the so-called only tool which was Palean boats had not been found, the author predicted that the Woven baskets could not have been taken from Palea to Lithos, an ancient village across the Brim River from Palea. Woven baskets might be from Lithos
开头段
According to the argument, the author believed that the so-called Palean baskets were not uniquely Palean just because they were found in another place across the Brim River from Palea. To strengthen his assumption, the author added that the Brim River is very deep and broad, so the ancient Paleans only had one method which is boating to cross the river. But no Palean boats had been found. There are several evidences such as the method to cross the river, the casual link between the boat and the Woven baskets which are needed to support the author’s assumption.
中间段
To begin with, the author believed that the river is deep and broad. But he did not give the exact number of the river’s depth and breadth. So we can not preclude that ancient Paleans can use other methods than boating to cross the river. Maybe they can swim across it or there is another way to go to the other side of the river.
In addition, the author said that no Palean boats had been found, but he did not instruct exactly the casual link between the boat and the Woven baskets. He also did not say the time of the  village of Palea and the village of Lithos. Maybe they were in different times and the woven baskets were taken to Lithos when there was no river.

作者: huan19880122    时间: 2017-8-17 23:56:07

Arg 1 Palean Basket unique


Woven baskets characterized by a particular distinctive pattern have previously been found only in the immediate vicinity of the prehistoric village of Palea and therefore were believed to have been made only by the Palean people. Recently, however, archaeologists discovered such a 'Palean' basket in Lithos, an ancient village across the Brim River from Palea. The Brim River is very deep and broad, and so the ancient Paleans could have crossed it only by boat, and no Palean boats have been found. Thus it follows that the so-called Palean baskets were not uniquely Palean.


Write a response in which you discuss what specific evidence is needed to evaluate the argument and explain how the evidence would weaken or strengthen the argument.


Claim 1:Palean Basket is only made by the Palean people

Data 1: Palean Basket had only been found in the immediate vicinity of Palea village

Warrant 1: the present known place that found the Palean Basket is the place it was made.

Evidence: More information about the archaeological progress in the nearby area around the Palea.

More information on the discovery of Palean Basket. Whether it is easily preserved even after the long history.

The Condition of Palea, whether it has the condition that allows the basket to preserve throughout the long history.



Claim 2: ancient Paleans could not cross the Brim River

Data 2: The Brim River is very deep and broad and there is no boat found

Warrant 2:

1. Brim River is as deep and broad in ancient times as today

2. There is no other way other than by boat to cross the Brim River

3. Ancient Paleans had no boat.

Evidence: The geographic information in ancient times of Palea and Brim River. The wether conditions in ancient times of Palea.


Claim 3: so-called Palean baskets were not uniquely Palean

Data3 :

1. archaeologists discovered such a 'Palean' basket in Lithos, an ancient village across the Brim River from Palea.

2. Claim2: ancient Paleans could not cross the Brim River

Warrant3: the basket could be brought to Lithos only by people.


首段

The argument states that the so-called Palean baskets may not be uniquely Palean as one had been found in a village from across a river. To evaluate the credibility of the argument conclusion, a lot more evidences are needed in order to illustrate as least the three following aspects: archaeological progress of Palean Basket, whether Palean baskets could be brought to Lithos only by people and whether Paleans could cross the Brim River.


中间段

The argument assumes if Palean basket is made by Palean people, its appearance in Lithos has to be brought over by people. However, many other ways could also do the work. The baskets has been preserved for thousands of years, a long time that plains could have become mountains. The basket might be buried in Palea however moved to Lithos along with geographical plate movement, and therefore were discovered there. It could also be brought by hurricanes, flood or simply drift over along Brim River. Without contradictory evidences, it could even be carried there by a boring bird, who stole the basket from Palea, flew to Lithos, and dropped it there.




作者: Ptisenbonandrab    时间: 2017-8-17 23:56:38

53)The following appeared in a health magazine.

The citizens of Forsythe have adopted more healthful lifestyles. Their responses to a recent survey show that in their eating habits they conform more closely to government nutritional recommendations than they did ten years ago. Furthermore, there has been a fourfold increase in sales of food products containing kiran, a substance that a scientific study has shown reduces cholesterol. This trend is also evident in reduced sales of sulia, a food that few of the most healthy citizens regularly eat.

Write a response in which you discuss what specific evidence is needed to evaluate the argument and explain how the evidence would weaken or strengthen the argument.

1
claim: their eating habits they conform more closely to government nutritional recommendations than they did ten years ago
data: a recent survey
warrant:the survey is not valid when its result is used for comparison between the current situation and the past
evidence: the survey has only be conducted recently

2
claim: there has been a fourfold increase in sales of food products containing kiran
warrant:the increased consumption of this kind of food may be due to various reasons
evidence: the population of the city has also increased a lot during the past few years and the food consumption per capital has decreased

3
claim: reduced sales of sulia, a food that few of the most healthy citizens regularly eat
warrant:There are other reasons for the reduction
evidence: the price of sulia has increased a lot recently

开头段
A recent report on a health magazine announces that citizens in Forsythe lead a healthier lifestyle than before. With a recent survey on people’s eating habits and the data of specific food sale in the market, the reporter draw the conclusion. However, several evidence is needed to validate the argument raised by the report.

第二段
There may be various reasons for the change in the consumption of kiran. To make sure the increase in kiran consumption is due to the improvement of people’s awareness in healthy problem, we should compare the kira consumption per capital instead of the total amount. Therefore, the data of the population of the city should be taken into the investigation when writing the report.
作者: Dover999    时间: 2017-8-17 23:57:59

53) The following appeared in a health magazine.

The citizens of Forsythe have adopted more healthful lifestyles. Their responses to a recent survey show that in their eating habits they conform more closely to government nutritional recommendations than they did ten years ago. Furthermore, there has been a fourfold increase in sales of food products containing kiran, a substance that a scientific study has shown reduces cholesterol . This trend is also evident in reduced sales of sulia, a food that few of the most healthy citizens regularly eat.

Write a response in which you discuss what specific evidence is needed to evaluate the argument and explain how the evidence would weaken or strengthen the argument.

1. Claim: The citizens of Forsythe have adopted more healthful lifestyles.

Data: Their responses to a recent survey show that in their eating habits they conform more closely to government nutritional recommendations than they did ten years ago.

Warrant: One’s healthy body can benefit from the  government nutritional recommendations

Evidence:
a). The data of food habits of people are needed and they should compare with the the  government nutritional recommendations more specifically.
b). The scientific evidences of  government nutritional recommendations.

2. Claim: The citizens of Forsythe have adopted more healthful lifestyles.

Data: Furthermore, there has been a fourfold increase in sales of food products containing kiran, a substance that a scientific study has shown reduces cholesterol .

Warrant: A healthy lifestyle has been adopted through increasing the eating of a wholesome food.
作者: justicekl    时间: 2017-8-17 23:59:48

53)The following appeared in a health magazine.

The citizens of Forsythe have adopted more healthful lifestyles. Their responses to a recent survey show that in their eating habits they conform more closely to government nutritional recommendations than they did ten years ago. Furthermore, there has been a fourfold increase in sales of food products containing kiran, a substance that a scientific study has shown reduces cholesterol. This trend is also evident in reduced sales of sulia, a food that few of the most healthy citizens regularly eat.

Write a response in which you discuss what specific evidence is needed to evaluate the argument and explain how the evidence would weaken or strengthen the argument.

23:22
In this argument, the author maintains that the citizens of Forsyth(F) have adopted more healthful lifestyles.  By referring to a study about the eating habits of F, the author tries to persuade us that they really live a good eating lifestyle.He also points out the sales of two kinds of food which affect the health to strengthen his view. Then, he try to sustain claim about the health lifestyle with only eating habits. However,regarding the unconvincing reasons provided by the author,we fail to reach a conclusion as he provides .

At first, the author consider about a recent study, which suggests that the eating habits are closer to government nutritional recommends than ten years ago to prove that the eating habit in F are getting progress today . However, more closely conforming to government than 10 years ago doesn’t means they have adopted the good eating habits. Maybe the government nutritional recommends are not true because the government just accepts the recommends from other areas and doesn’t not make serious examination into it. Thus the so-call nutritional recommends might be not that suitable for people in F. Even though the government provides the nutritional recommendations that is exactly suitable for domestic people, the study just suggests their eating habits are more closely conform to recommendations than 10 years ago rather than every past 10 years. That comes up with some skeptical problem because we don’t know if the people’s eating habit are becoming more and more consistent with the recommendations year by year. Some possibilities may occur that the eating habits are more away from that of the government suggests even though closer to 10 years ago.Too little information are produced in this study that we can’t regard it a good proof for the author has underlined.

In conclusion,it is difficult to confirm that the lifestyles in F are getting better now. The assumptions discussed above need to be critically examined taking into account more detailed information in issues such as the study of the extent of the consistence of peop
作者: 芸乔    时间: 2017-8-18 00:10:21

Arg 1 Palean Basket unique

Woven baskets characterized by a particular distinctive pattern have previously been found only in the immediate vicinity of the prehistoric village of Palea and therefore were believed to have been made only by the Palean people. Recently, however, archaeologists discovered such a 'Palean' basket in Lithos, an ancient village across the Brim River from Palea. The Brim River is very deep and broad, and so the ancient Paleans could have crossed it only by boat, and no Palean boats have been found. Thus it follows that the so-called Palean baskets were not uniquely Palean.

Write a response in which you discuss what specific evidence is needed to evaluate the argument and explain how the evidence would weaken or strengthen the argument.


claim: woven baskets were not only made by the Palean people
data: a "Palean" basket in Lithos, no Palean boats
warrant: Brim River is deep and broad and people could not crossed it only by boats

开头段:
The author of this article argues that the woven baskets were not uniquely Palean for it was not only found in the prehistoric village of Palea but also in Lithos. To support the argument, the author proclaims that Palea and Lithos are divided by the broad and deep Brim River, which could be crossed only by boat. Yet boats have not been found in Palea so far. Although the presence of a Palean basket in Lithos may lead to a presumption that Lithos also produced the baskets, more evidence is needed to be found to access the argument.

中间段:
Although it is found that the deep and broad Brim River could have been crossed only by boat, the geologic transformation should be further discovered or explained by scholars to help us evaluate the authors assumption. The width of the river could have changed from the past to present. If the Brim River was broad and inaccessible to cross in the time the ancient Palean people lived, the argument would be strengthened. However, if the River was shallow and did not exist at that time, the author's assertion would not be solid at all.

作者: sofresh666888    时间: 2017-8-18 00:12:13

1)
Woven baskets characterized by a particular distinctive pattern have previously been found only in the immediate vicinity of the prehistoric village of Palea and therefore were believed to have been made only by the Palean people. Recently, however, archaeologists discovered such a 'Palean' basket in Lithos, an ancient village across the Brim River from Palea. The Brim River is very deep and broad, and so the ancient Paleans could have crossed it only by boat, and no Palean boats have been found. Thus it follows that the so-called Palean baskets were not uniquely Palean.

Write a response in which you discuss what specific evidence is needed to evaluate the argument and explain how the evidence would weaken or strengthen the ar

Claim: Woven baskets were only made by Palean people.  
Data: Woven baskets characterized by a particular distinctive pattern have previously been found only in the immediate vicinity of the prehistoric village of Palea and therefore were believed to have been made only by the Palean people.
Evidence: Palea is the only place which woven baskets have been found.
Question: Which year/century do the baskets from? What type of character do the baskets have? If they are produced in different years, probably it was handed down generations by generations.

Claim: Other place has ‘Palean’ basket as well.
Data: Recently, however, archaeologists discovered such a 'Palean' basket in Lithos, an ancient village across the Brim River from Palea.
Evidence: Woven baskets also know as ‘Palean’ baskets have been found in Lithos as well.
Question: what was the function of the baskets in Lithos/Palean people’s? Were the function same in two ancient village?
Evidence:  A. a very common container in both Lithos & Palean people’s daily life.
         B. a specific tool
         C. different function in different village   


Claim: Paleans didn’t bring the basket through Brim River. The baskets are originally from Lithos.
Data: The Brim River is very deep and broad, and so the ancient Paleans could have crossed it only by boat, and no Palean boats have been found.
Evidence: No evidences show that Paleans had been to Lithos.
Assumption: 1. Besides boat, other ways of transportation are also available to pass the baskets. 2. With the same starting material, it is very possible that two villages could produce similar type of items.

Claim: Palean baskets can be made by other too.
Data: Thus it follows that the so-called Palean baskets were not uniquely Palean.
Evidence: Palean baskets are not only made by Palean.

Introductory:
According to this argument, the author believes that ‘Palean’ baskets are not only made by Paleans but also other people from different village. By finding the baskets in other village, the author assumes that Palea is not the only place that produces Woven baskets. A number of assumptions have been made in the argument by stating that the same type of baskets have been found in Lithos, however, no boat existing evidence shows Palean had been there. These assumptions need to be scrutinized critically to assess the strength of the argument.

Middle 1st:
According the to archeologist’s discovery in Lithos, ‘Palean’ baskets have been found in there as well. By this finding, the author believes that since Brim River is deep and board enough, Palean boat could have passed through it and reached to Litho. However, as the matter of fact there are no evidences show Palean boat had been to here. Therefore, the author assumes that Litho also might ‘Palean’ baskets’ producing place.
作者: Margueritexy    时间: 2017-8-18 00:14:28

Arg 1 Palean Basket unique

Woven baskets characterized by a particular distinctive pattern have previously been found only in the immediate vicinity of the prehistoric village of Palea and therefore were believed to have been made only by the Palean people. Recently, however, archaeologists discovered such a 'Palean' basket in Lithos, an ancient village across the Brim River from Palea. The Brim River is very deep and broad, and so the ancient Paleans could have crossed it only by boat, and no Palean boats have been found. Thus it follows that the so-called Palean baskets were not uniquely Palean.

Write a response in which you discuss what specific evidence is needed to evaluate the argument and explain how the evidence would weaken or strengthen the argument.

Claim: The ancient Paleans could have crossed it only by boat.
Data: The Brim River is very deep and broad.
Warrant: The ancient Paleans could cross the river only by boat because the river is so deep and broad.
Evidence needed:
1.Information about other possible ways for Paleans to crossing the river——bridge, some connections of some parts
2. Is there geographic change from the ancient tonow?

Claim: There was no boat for Palean to cross the river.
Data: no Palean boats have been found
Warrant: The basket couldn’t be sent through the river by Palean
Evidence needed:
1.        some parts of the two sides of river were connected
2.        Lithos might go through the river and carry baskets away

According to the author, the Palean baskets were not uniquely Palean because archaeologists found a so-called Palean basket in Lithos across the Brim River from Palea and they believed that there were no ways to bringing the basket cross the river. However, there are some assumptions made in the argument to support the result, we need more evidence about the geographic change in the past, ways to crossing the river and some particularity in geographic condition.

In conclusion, it is difficult to confirm that the palean basket is not uniquely Palean. The possibility and assumptions discussed above could reduce the certainty and the strength of the argument. Thus, we need to take into account more relevant factors on the argument such as the geographic condition and change from the ancient to the current, the other possible ways to help the connection between Palean and Lithos and some other possibilities.

作者: telepathyivan    时间: 2017-8-18 01:11:21


Arg 1 Palean Basket unique

Woven baskets characterized by a particular distinctive pattern have previously been found only in the immediate vicinity of the prehistoric village of Palea and therefore were believed to have been made only by the Palean people. Recently, however, archaeologists discovered such a 'Palean' basket in Lithos, an ancient village across the Brim River from Palea. The Brim River is very deep and broad, and so the ancient Paleans could have crossed it only by boat, and no Palean boats have been found. Thus it follows that the so-called Palean baskets were not uniquely Palean.

Write a response in which you discuss what specific evidence is needed to evaluate the argument and explain how the evidence would weaken or strengthen the argument.
                                                                                                                                          

Claim: the so-called Palean baskets were not uniquely Palean.
Data: archaeologists discovered such a 'Palean' basket in Lithos, an ancient village across the Brim River from Palea. The Brim River is very deep and broad, and so the ancient Paleans could have crossed it only by boat, and no Palean boats have been found.
Warrant: The discovery of Palean basket in Lithos, the deep river between these two village and no remains of boat substantiate that the basket is not uniquely made in Palean
Evidence:
Whether the river was as deep as now or even existed
Whether these two villages interacted before through other villages
What type of material made the boat there can it survived the erosion of time

Intro paragraph
In this argument the author argues that since the Palean basket that is originally thought to exist only in one place was found in Litho, the so-called Palean basket was not unique to be Palean. To support this argument, the author emphasizes that the Brim river between these two places is deep and can only be crossed by boat, and noted that there is no trace of boat. Although the purported Palean basket may indeed be made there, more evidence about the river’s history, trade between villages and the material of boat is needed to help evaluate the real origin of this basket.

Mid paragraph
To begin with, evidence revealing the history of the Brim river is needed to help the evaluation of the author’s assertion that the river has always been deep and broad and could only be crossed by boat. However, this does not receive any support in the argument and could possibly be wrong since the earth’s geological changes happen all the time. Record of the river and the analysis of the sediment laying deep in the river can shed light on our judgement. If the evidence turns out that the river used to be shallow or did not exist at all, then the author’s argument is undoubtedly weakened. Otherwise, it is strenghtened.

作者: 芥弥    时间: 2017-8-18 01:43:27

151) Benton City residents have adopted healthier lifestyles. A recent survey of city residents shows that the eating habits of city residents conform more closely to government nutritional recommendations than they did ten years ago. During those ten years, local sales of food products containing kiran, a substance that a scientific study has shown reduces cholesterol, have increased fourfold, while sales of sulia, a food rarely eaten by the healthiest residents, have declined dramatically. Because of these positive changes in the eating habits of Benton City residents, we predict that the obesity rate in the city will soon be well below the national average.

Write a response in which you discuss what questions would need to be answered in order to decide whether the prediction and the argument on which it is based are reasonable. Be sure to explain how the answers to these questions would help to evaluate the prediction.

A recent survey of city residents shows that the eating habits of city residents conform more closely to government nutritional recommendations than they did ten years ago.
Claim: Benton City residents have a healthier life than before.
Data: A recent survey
Warrant: Conformity to the government nutritional recommendations means a healthier lifestyle.
Question: Representativeness of the survey
- Standard of the survey
- Details of government nutritional recommendations

During those ten years, local sales of food products containing kiran, a substance that a scientific study has shown reduces cholesterol, have increased fourfold, while sales of sulia, a food rarely eaten by the healthiest residents, have declined dramatically.
Claim: Benton citizens' diet becomes healthful.
Data: Local sales of healthy food products have increased, while the less healthy ones have decreased.
Warrant: More sales of health food represents a healthier life.
Question: Other factors influencing local sales
- Nature of kiran and sulia

Because of these positive changes in the eating habits of Benton City residents, we predict that the obesity rate in the city will soon be well below the national average.
Claim: Obesity rate in the city will decrease since people have a good eating habit.
Data: Estimation
Warrant: A balanced eating habit will lead to obesity decrease.
Question:Other factors influencing obesity

Introduction:
According to the letter, residents in Benton City have achieved a balanced diet, thus leading to the decrease of obesity rate below the national average in the near future. This projection is based on a survey of local eating habits and a report of local sales of food. We have to ask a few questions about the recent survey, the government nutritional recommendation and local food sales In order to determine the soundness of this argument.

One paragraph:
Given the positive changes in residents eating habits, we still need to ask the question about the cause of obesity. It is true that a healthy and balanced diet would be conducive to decrease obesity rate. However, it is important to bear in mind that there is a host of factors leading to obesity and an unbalanced diet is only a part of it. Maybe genetic factor plays an important role in most of the obesity cases in Benton. As a results, even if people adopt a healthful eating habit, their hereditary obesity would not be cured simply with a salubrious diet. Therefore, in order to decide whether this positive change could decrease local obesity rate, we should collect more information about the possible factors triggering obesity and question the necessary link between the diet and the obesity rate.
作者: Herio_HKeKe    时间: 2017-8-18 02:09:36

53) The following appeared in a health magazine.

The citizens of Forsythe have adopted more healthful lifestyles. Their responses to a recent survey show that in their eating habits they conform more closely to government nutritional recommendations than they did ten years ago. Furthermore, there has been a fourfold increase in sales of food products containing kiran, a substance that a scientific study has shown reduces cholesterol. This trend is also evident in reduced sales of sulia, a food that few of the most healthy citizens regularly eat.

Write a response in which you discuss what specific evidence is needed to evaluate the argument and explain how the evidence would weaken or strengthen the argument.

claim:Forsythes' people have a healthier lifestyle
data:a recent survey show that in their eating habits they conform more closely to government nutritional recommendations than they did ten years ago
warrant:the more closely to government nutritional recommendations means the healthier diet they had which is a part of healthy lifestyle
assumption: the survey is categorical precise
               - the number of people investigated is too small to reflect the authentic situation
assumption:be close to the nutritional recommendation means health nutritional intake
               - even people's diet be more closely to the recommendations, it may just means the eating habits                  turn back to normal condition since the former ones of locals is unhealthy

claim: Forsythes' people have a healthier lifestyle
data: here has been a fourfold increase in sales of food products containing kiran
warrant: kiran is a substance that a scientific study has shown reduces cholesterol
assumption: people buy food products with kiran because kiran is healthy
               - the reason why people consume those food products may just is the taste of them rather than the function of diminishing cholesterol.
               - the company which manufacture those products has discounted them to spur demand.

claim: Forsythes' people have a healthier lifestyle
data: sales of sulia reduced
warrant:  sulia is a food that few of the most healthy citizens regularly eat
assumption: decrease in sales of sulia  means increase in the most healthy citizens
               - it could be the futile harvest or increasing price of sulia results to reduction in sulia's sales.
assumption: the most healthy citizens doesn't eat sulia regularly because sulia is unhealthy.
               - normal healthy people still eat sulia
               - frequency of eating sulia also depends on how hard cook it could be and how busy people are. not    regular does't means never.

Intro:
According to the health magazine, local people in Forsythe live more healthier than before. To substanciate the conclusion, the magazine points out evidence that a survey result about changes of locals' daily diet. Furthermore, it also indicates the trends of sales of specific food. These evidences need to be scrutinized critically to assess the strength of the argument.

TS2:
On the other hand, the magazine provides the trends of sales of kiran-products and sulia to explain the healthier changes in residents' lifes. However, the sales changes of these items just can represent a little locals' consciousnesses about health as it can be influenced by numerous factors. Besides kiran's healthy function of diminishing cholesterol, for example, the increasing sales of food products containing kiran may also depend on the ambrosial taste of them or just the discounted price, which cannot evince that citizens have a healthier food choice. And no matter sulia is healthy food or not, the reduction in sales of sulia could also attribute to futile harvest of it. In addition, the definition of sulia is vulnerable because the frequency of eating sulia also depends on how hard cooking it could be and how busy people are. Unless eliminate these irrelevant interferences, the sales of food including its products cannot effectively back this argument.
作者: Ibelieving    时间: 2017-8-18 03:15:32

144
The citizens of Forsythe have adopted more healthful lifestyles. Their responses to a recent survey show that in their eating habits they conform more closely to government nutritional recommendations than they did ten years ago. Furthermore, there has been a fourfold increase in sales of food products containing kiran, a substance that a scientific study has shown reduces cholesterol. This trend is also evident in reduced sales of sulia, a food that few of the healthiest citizens regularly eat.

Write a response in which you examine the stated and/or unstated assumptions of the argument. Be sure to explain how the argument depends on these assumptions and what the implications are for the argument if the assumptions prove unwarranted.

Claim:The citizens of Forsythe have adopted more healthful lifestyles.
Data:eating habits are healthier.
Warrant:eating healthy means a healthy lifestyle.

Assumption:healthy Eating habits are the most important thing in our life.

Claim:eating habits are more healthy than before(implied)
Data1:Their responses to a recent survey show that in their eating habits they conform more closely to government nutritional recommendations than they did ten years ago
Warrant1:government nutritional recommendations mean healthy eating habits.

Assumptions1:government nutritional recommendations are healthier than personal diet.

Data2:there has been a fourfold increase in sales of food products containing kiran, a substance that a scientific study has shown reduces cholesterol
Warrant2:more food containing kiran indicate that people hava healthier eating habits.

Assumption2: the population of the city keep stable in the past ten years.

Data3: This trend is also evident in reduced sales of sulia, a food that few of the healthiest citizens regularly eat.
Warrant3:what few of the healthiest citizens usually eat are healthy

Assumption3:that healthiest citizens refuse to eat are not healthy.

开头段:
According to the author of the passage,citizens of Forsythe now live a healthier life compared with that ten years ago, based on the fact that the citizens’eating habits are conform to government recommendations,purchase more kiran-rich food and keep away from what healthiest people seldom buy.A number of assumptions have been made about these facts and need to be scrutinized critically to assess the strength of the argument.

第一段:
To begin with, it is assumed that eating in a healthy way means having a healthy lifestyle.While the citizens are doing well in diet,it does not  follow that they have necessarily paid enough attention to other aspects in life.Maybe a large quantity of the citizens are suffering heavy work load and lack of sleep, either of which will contribute to latent disease.
Unless more surveys are available to suggest that the citizens have actually been healthier except for eating habits, the assumptions made here do not hold.
作者: flhope    时间: 2017-8-18 10:56:11

1)Woven baskets characterized by a particular distinctive pattern have previously been found only in the immediate vicinity of the prehistoric village of Palea and therefore were believed to have been made only by the Palean people. Recently, however, archaeologists discovered such a 'Palean' basket in Lithos, an ancient village across the Brim River from Palea. The Brim River is very deep and broad, and so the ancient Paleans could have crossed it only by boat, and no Palean boats have been found. Thus it follows that the so-called Palean baskets were not uniquely Palean.

Write a response in which you discuss what specific evidence is needed to evaluate the argument and explain how the evidence would weaken or strengthen the argument.

Claim: The so-called Palean baskets were not uniquely Palean.
Data: Archaeologists discovered such a 'Palean' basket in Lithos, an ancient village across the Brim River from Palea.
Warrant: The baskets found in Lithos were local product.
Evidence needed: People in Lithos are able to produce such kind of basket.

Claim: it’s impossible for the Paleans to cross the river
Data: The Brim River is very deep and broad and no Palean boats have been found
Warrant: the ancient Paleans could have crossed it only by boat
Evidence needed: 1. the river is deep and broad in the ancient times.
                          2. the boats that could be used to cross the river never existed

Claim: Woven baskets were believed to have been made only by the Palean people.
Data: Woven baskets characterized by a particular distinctive pattern have previously been found only in the immediate vicinity of the prehistoric village of Palea.
Warrant: Palean people made these woven baskets.
Evidence needed: Palean people are isolated.


开头段:
In the argument, it is concluded that the so-called Palean basket were not uniquely Palean. Firstly, recent discovery of a ‘Palean’ basket in Lithos represents that the woven baskets are not uniquely Palean. Secondly, because of the deep and broad river between these two villages and on boats have been found, the basket found in Lithos is impossible to be made in Palea. While the conclusion could be true, close scrutiny reveals that the conclusion lacks critical support and therefore we need more evidence to help evaluate the argument.

中间段:
In addition, we need more evidence to ascertain that baskets found in Lithos were not delivered from Palea. There are two facts given to support the claim: the river between these two villages is too broad and deep to cross without boat and no Palean boats have been found. However, the reasoning process of these facts are not convincing. For example, it’s possible that, in ancient times, the Brim river was not wide and deep as we see today which would be easy for the Palean to cross. Also, the non-existence of boats at present could be attributed to the reason that these boats have decayed during the long period. Thus, we need evidence of the absolute condition of Brim river in ancient times and of the existence of boats at that time to re-examine the claim.

作者: 猜cc    时间: 2017-8-18 12:31:18

144) The citizens of Forsythe have adopted more healthful lifestyles. Their responses to a recent survey show that in their eating habits they conform more closely to government nutritional recommendations than they did ten years ago. Furthermore, there has been a fourfold increase in sales of food products containing kiran, a substance that a scientific study has shown reduces cholesterol. This trend is also evident in reduced sales of sulia, a food that few of the healthiest citizens regularly eat.

Write a response in which you examine the stated and/or unstated assumptions of the argument. Be sure to explain how the argument depends on these assumptions and what the implications are for the argument if the assumptions prove unwarranted.

Claim: The citizens of Forsythe have adopted more healthful lifestyles.
Data: survey; food sales.
Assumption:  surveys about people’s eating habits and food sales reveal exactly the lifestyles of citizens of Forsythe.

Claim: Their responses to a recent survey show that in their eating habits they conform more closely to government nutritional recommendations than they did ten years ago.
Data: responses of the citizens to a survey.
Warrant: survey shows the eating habits of citizens is healthier, compared with ten years ago.
Assumptions:
Responses of the survey perfectly shows what respondents think about.
Responses of the survey perfectly represent all the citizens.
The questions concluded in the survey shows exactly the eating habits of citizens.
The questions asked in the survey reveals exactly the governmental recommendation.
The factor used to represent the degree of ten years ago is valid.

Claim: Furthermore, there has been a fourfold increase in sales of food products containing kiran, a substance that a scientific study has shown reduces cholesterol.
Data: sales of food products containing Kiran, a healthy elements.
Warrant: increase of healthy food sales prove citizens’ lifestyles are more healthy.
Assumptions:
the increase of food sales means citizens buy more.
the way of cooking can preserve kiran’s function.

Claim: This trend is also evident in reduced sales of sulia, a food that few of the healthiest citizens regularly eat.
Data: food sales
Warrant: decrease in the sales of food, which the healthy people hardly eat, shows people’s lifestyles are more healthy.
Assumptions:
The healthiest citizens’ dietary habits shows whether food is healthy.
Reduced sales means citizens buy less.
下午补上开头段和中间段。
作者: 心玉夕颜    时间: 2017-8-18 13:39:49

53) The following appeared in a health magazine.

The citizens of Forsythe have adopted more healthful lifestyles. Their responses to a recent survey show that in their eating habits they conform more closely to government nutritional recommendations than they did ten years ago. Furthermore, there has been a fourfold increase in sales of food products containing kiran, a substance that a scientific study has shown reduces cholesterol. This trend is also evident in reduced sales of sulia, a food that few of the most healthy citizens regularly eat.

Write a response in which you discuss what specific evidence is needed to evaluate the argument and explain how the evidence would weaken or strengthen the argument.


claim: The citizens of Forsythe have adopted more healthful lifestyles.

data:
1)Their responses to a recent survey show that in their eating habits they conform more closely to government nutritional recommendations than they did ten years ago.
2)There has been a fourfold increase in sales of food products containing kiran.
3) Reduced sales of sulia, a food that few of the most healthy citizens regularly eat.

warrant:
1) A recent survey can show the change of eating habits during ten years.
2)&3) Increase in sales of food products containing kiran and reduction in sulia mean that more and more people choose to eat healthy food.

Evidence:
1) The number of respondents of that recent survey or how much the percentage of them of all citizens of Forsythe.
2) The kinds of food sold in markets is largely the same as ten years ago. Since the government announced nutritional recommendations, the food industries may change their unhealthy products into more healthy ones.
3) The time of the data that the change in sales of food compared by the author is unclear.
4)The quantity of supply of food contain kiran and sulia during ten years is unclear. Because maybe ten years ago, food contain kiran supplyed in Forsythe were much fewer than now and sulia were more than now which can also results the different trends of sales of food.



开头段:
The magazine concludes that the citizens of Forsythe have live healthier. The author shows three pieces of evidence, a recent survey and trends of food in these years,  in order to support the conclusion. These evidence might seem convincing and valid at first glance, however, it is not logical as the author expects. In order to thoroughly evaluate this essay, more additional evidence is needed.


中间段:
The first piece of evidence needed to evluate the validity of that survey is how many people of Forsythe were interviewed and what proportion of these respondents to all residents in Forsythe.
Since the scale of this survey is unknown, it is unpersuasive to assert that citizens of Forsythe have adopted more healthful lifestyles.
作者: TaylorW    时间: 2017-8-18 17:29:50

xiulin2015 发表于 2017-8-17 22:57
your middle paragraph is impressive as it is a holistic review on the research structure and how t ...

I didn't except any instructions for my essay for it is flooded by others quite fast. So thank you, thank you for your advice it’s much needed. I feel like to point out the assumptions by the order of the paragraph, since the survey is the first factor mentioned in the article so I just put it at the very beginning .I’ll give it a thought later. And thank u for mention the evidence thing, I totally forgot it. yep! they should provide blood test data or citizen’s shopping list to make sure they tell the truth…
作者: John-science    时间: 2017-8-19 21:20:03

53) The following appeared in a health magazine.

The citizens of Forsythe have adopted more healthful lifestyles. Their responses to a recent survey show that in their eating habits they conform more closely to government nutritional recommendations than they did ten years ago. Furthermore, there has been a fourfold increase in sales of food products containing kiran, a substance that a scientific study has shown reduces cholesterol. This trend is also evident in reduced sales of sulia, a food that few of the most healthy citizens regularly eat.

Write a response in which you discuss what specific evidence is needed to evaluate the argument and explain how the evidence would weaken or strengthen the argument.

1
Claim:   the citizens of Forsythe have adopted more healthful lifestyles

Date:    a recent survey show that in their eating habits they conform more closely to government nutritional recommendations than they did ten years ago

Warrant: eating habit which is closed to the government nutritional recommendations is more healthier.

Evidence: 1-the result of survey is representative and valid.
         2- the government nutritional recommendations is suitable for every citizen.

2
Claim: The citizens of Forsythe have adopted more healthful lifestyles
Date: there has been a fourfold increase in sales of food products containing kiran which help reduces cholesterol
Warrant: the increasing sale of kiran can help people reduces cholesterol which benefit human health.
Evidence: 1- the scientific study result is wildly accepted and valid.
        2- the scientific study is carried out taken human as example, shown eating kiran can help reduces cholesterol.
        3- The number of population keep constant.
        4- selling kiran is used by civics
3
Claim: The citizens of Forsythe have adopted more healthful lifestyles\
Date:  trend of reduceing sales of sulia which is a food most healthy people don’t eat.
Warrant:  reducing sales of sulia making people’s eating habit more healthy.
Evidence: 1 the trend is representative and valid. or The origin source of this information is representative and valid
         2 eating sulia is bad for everyone health.
         3 reduction in selling of sulia means that less people eat it.
Introductory paragraph:
As the statement in a health magazine shows that the citizens of Forsythe is leading a more healthy lifestyles than before. Due to the eating habits which is more closely to the one government recommends, increasing selling of kiran which can reduce cholesterol and lower selling amount of sulia which most healthy people don’t eat, the magazine concluded that those facts verify the more healthy citizen lead. However, more extra compelling and comprensive evidences about survey of eating habit, kiran and sulia sales are needed to support the the argument.

Middle paragraph:
First of all, the magazine assumed that the recently survey which shows a increasing similarity between the eating habit of citizens and the one authority recommends can reflect the more healthy lifestyles people lead. Though the survey exhibited that civic eating habit became more similar to government nutritional recommendations, it doesn’t indicate that citizens of Forsythe have adopted more healthful lifestyles. Maybe the survey result is only based on few people and therefore is not representative and valid. Also the government nutritional recommendations can’t suit to every citizen, the amount meat which old people and teenager should eat is totally diffident. So extra evidences should be given to support the representative of survey and valid of government nutritional recommendations in order to draw the conclusion that citizens of Forsythe have adopted more healthful lifestyles

作者: ning97    时间: 2017-8-21 23:45:41

Arg 1 Palean Basket unique

Woven baskets characterized by a particular distinctive pattern have previously been found only in the immediate vicinity of the prehistoric village of Palea and therefore were believed to have been made only by the Palean people. Recently, however, archaeologists discovered such a 'Palean' basket in Lithos, an ancient village across the Brim River from Palea. The Brim River is very deep and broad, and so the ancient Paleans could have crossed it only by boat, and no Palean boats have been found. Thus it follows that the so-called Palean baskets were not uniquely Palean.

Write a response in which you discuss what specific evidence is needed to evaluate the argument and explain how the evidence would weaken or strengthen the argument.

Claim1: Woven baskets were believed to have been made only by the Palean people.
Data: Woven baskets characterized by a particular distinctive pattern have previously been found only in the immediate vicinity of the prehistoric village of Polea.
Warrant: Baskests were just decoveried in one place, so that only made in this place.

Evidence needed: 1) There is nothing about woven products(not only baskets) were found. Or even more specificly, the Palean people made the woven baskets.
                          2) more information such about the woven products introduced into the Palean people's life.


Claim 2:Thus it follows that the so-called Palean baskets were not uniquely Palean.
Data: archaeologists discovered such a 'Palean' basket in Lithos, an ancient village across the Brim River from Palea.
Warrant:The Brim River is very deep and broad, and so the ancient Paleans could have crossed it only by boat, and no Palean boats have been found.
Evidence needed: 1) there is no other ways to cross the river, such as bridge, the rocks.
                          2) the river was as big as it is now.
                          3) there is no other influences such as earthquake, wind, or even flu across the river.

Claim 3: There is no communications between the Palean people and the Lithos' people.
Data: The Palean people can't talk or teach Lithos' people, so that they make the woven baskets by themselves.
Evidencen needed: Any historical documents which first reported the two villages.

Intro progress:
According to the 'Palean' basket discovered by archaeologists in Lithos, an ancient village across the deep and broad Brim River, the argument drew to a conclusion that so-called Palean baskets were not uniquely Palean. However, the deduction of the conclusion is not sound unless several assumptions, which means we need more evidence to conclude.
TS1
First, the woven baskets discovered in the Palea must be artificated by the Palean people, not learning from others. Though we have evidence that the woven baskets were discovered firstly in the Palea, there still are many other possiblities, such as they learn from other traders even may came from Lithos. The perfect evidence is the  literaly telling the process how the woven could be made into baskets occured to Palean people.  Besides that, the appropriate years the baskets were made also helps us decide who created the woven baskets first.
作者: 夏季的热度    时间: 2017-8-25 06:40:02

53) The following appeared in a health magazine.

The citizens of Forsythe have adopted more healthful lifestyles. /Their responses to a recent survey show that in their eating habits they conform more closely to government nutritional recommendations than they did ten years ago. /Furthermore, there has been a fourfold increase in sales of food products containing kiran, a substance that a scientific study has shown reduces cholesterol. /This trend is also evident in reduced sales of sulia, a food that few of the most healthy citizens regularly eat.

Write a response in which you discuss what specific evidence is needed to evaluate the argument and explain how the evidence would weaken or strengthen the argument.
………………………………………..
Claim: The citizens of Forsythe have adopted more healthful lifestyles.

Claim:   The citizens of Forsythe have adopted more healthful lifestyles.
Data:     Their responses to a recent survey show that in their eating habits they conform more closely to government nutritional recommendations than they did ten years ago.
Warrant:  The survey accurately reflects the result.
Evidence: Need more evidence  about the survey to decide wether the survey can support the claim. ~survey/ eating habit/ sampling issue.
-Survey:how the survey is conducted; how are the questions designed; are those questions reliable and without any implied directions;
-Eating habits: did the survey ask the respondences their eating habits ten years ago as well as now? how did the result of the eating habit come from? is it from the data of old survey or from the  recalling of respondences? for the former, is the old survey reliable? for the latter one, more information need to be shown~
-Sampling issue:  region, age, gender
are the targeted respondents representative to the citizens of Forsythe,or are they just from a particular region?  did these respondences cover all age group, or just one or two age groups? are people in two genders equally responded or one gender outnumber another? because people in two genders may vary in diet habits.

Claim:    The citizens of Forsythe have adopted more healthful lifestyles.
Data:      there has been a fourfold increase in sales of food products containing kiran, a substance that a scientific study has shown reduces cholesterol.
Warrant:  the rapid increase of food products containing Kiran means people’s lifestyles are more healthful.
Evidence:  More information is needed to understand the connection of fourfold increase of food products containing Kiran and more healthful lifestyle.
  ~ kiran/ the food product/ why sales of the food products increase...
-Kiran reduces cholesterol. Kiran is healthful. Fourfold increase of kiran in food is healthful.>>their connection needs more evidences to prove. maybe it is not Kiran but other factors such as good moods that boost people to buy this kind of food, and they feel good, but not actually as good as they imagined.
-food containing Kiran is mainly consumed by healthful people. people who do not consume food containing kiran are less healthful. maybe the kiran food products are mainly brought by less healthful people rather than healthful people. or maybe both kinds of people like it, and it does not have really connections to people’s lifestyle.
-that sales of food products increase may because of other factors, such as market promotion, new favor fashions, failure of the competitors,etc. if it is the case, the survey data may not support the claim.

Claim:      The citizens of Forsythe have adopted more healthful lifestyles.
Data:       This trend is also evident in reduced sales of sulia, a food that few of the most healthy citizens regularly eat.
Warrant:   Reduced sales of sulia means most people take more healthful lifestyle.   
Evidence:  more information is required
- if less sulia means being healthful
-  if reduced sales means that less people eat it or because of other factors(market competition)
-  the reason why most healthy people do not eat it is because it is not healthy, not because it just tastes bad, no one likes it.
-  people are healthy due to their exercises or healthy conditions or good moods, do not necessarily have connections to the eating diet or one particular food.
……………………………………………………..
First paragraph:
[summarize the main conclusion]/[refer to the evidence]/[follow the writing instructions]/[develop the framework of this essay]/[a generic version of response to the writing instructions(better to customize)]

According to a health magazine, the citizens of Forsythe have adopted more healthful lifestyle. Citing a recent survey data and two sales of two types of food, the magazine jumps to such a claim. However, to justify it, we need more information about how the survey was conduced and designed and the reasons why the sales of the two types of food would have changed. These evidences needs to be provided to access to the strength of the argument.

3rd body paragraph:
[ point out the evidence to be needed]/
[ evaluate the evidence]/
[ explain how the evidence would weaken or strength the argument]

Furthermore, we need more information about the sales of sulia. According to the magazine, most healthy people barely eat sulia. But does that mean people are healthy because they do not eat sulia? That people are healthy may have nothing to do with their eating habit or one particular food such as sulia. The reason why they are healthy may just because they do exercise regularly into their daily routine and have good moods, etc.  We need more evidence about the amount of sulia to determine whether having less sulia is healthful. Also we need to consider the sales of sulia. It could be caused be factors that are completely unrelated to people’s lifestyle at all. Maybe the reduced sales is due to the market competition. Maybe sulia as a type of raw material goes high in price, so less and less people buy it and turn to its substitute. If this is the case, the data may not support the claim. Also, we need to further evaluate wether the substitute is healthful to people or not.





欢迎光临 寄托家园留学论坛 (https://bbs.gter.net/) Powered by Discuz! X2